Vous êtes sur la page 1sur 531

Board

 Refresher:    
Internal  Medicine    
Jaime  Alfonso  M.  Aherrera,  MD  DPCP  
Jose  Eduardo  DL.  Duya,  BSPH,  MD,  DPCP  
1.  You’ve  been  seeing  Mrs.  Reyes,  42/F  at  the  
OPD  for  chronic  heart  failure  and  she  has  been  
minimally  symptomaLc  on  a  beta-­‐blocker.  She  
comes  back  to  you  with  acute  decompensated  
heart  failure.  Which  of  the  following  might  have  
triggered  her  decompensaLon?  

 A.  Alcohol  abuse  


 B.  Smoking  
 C.  Pregnancy  
 D.  Fluid  restricLon  
1.  You’ve  been  seeing  Mrs.  Reyes,  42/F  at  the  
OPD  for  chronic  heart  failure  and  she  has  been  
minimally  symptomaLc  on  a  beta-­‐blocker.  She  
comes  back  to  you  with  acute  decompensated  
heart  failure.  Which  of  the  following  might  have  
triggered  her  decompensaLon?  

 A.  Alcohol  abuse  


 B.  Smoking  
 C.  Pregnancy  
 D.  Fluid  restricLon  
found to be helpful, particularly in patients with advanced disease.

Table 234-4 Factors That May Precipitate Acute Decompensation in


Patients with Chronic Heart Failure

Dietary indiscretion

Myocardial ischemia/infarction

Arrhythmias (tachycardia or bradycardia)


Discontinuation of HF therapy

Infection
Anemia

Initiation of medications that worsen HF

Calcium antagonists (verapamil, diltiazem)


Beta blockers

Nonsteroidal anti-inflammatory drugs


Antiarrhythmic agents [all class I agents, sotalol (class III)]
Anti-TNF antibodies
Alcohol consumption
Pregnancy
Worsening hypertension
Acute valvular insufficiency
2.  You  remember  that  Mrs  Reyes  has  had  
rheumaLcheart  disease  since  she  was  25  years  old.    
On  examinaLon,  she  has  an  accentuated  S1,  a  
diastolic  rumble  over  the  apex  and  a  wide,  notched  
P  wave  on  12L  ECG.  Which  valvular  lesion  does  she  
most  probably  have?  
A.  Mitral  stenosis  
B.  Mitral  regurgitaLon  
C.  AorLc  stenosis  
D.  AorLc  regurgitaLon  
2.  You  remember  that  Mrs  Reyes  has  rheumaLc  
heart  disease  since  she  was  25  years  old.    On  
examinaLon,  she  has  an  accentuated  S1,  a  diastolic  
rumble  over  the  apex  and  a  wide,  notched  P  wave  
on  12L  ECG.  Which  valvular  lesion  does  she  most  
probably  have?  

A.  Mitral  stenosis  


B.  Mitral  regurgitaLon  
C.  AorLc  stenosis  
D.  AorLc  regurgitaLon  
Mitral  Stenosis  (MS)  
•  Most  common  valvular  lesion  in  RHD  
•  RheumaLc  heart  disease  is  the  leading  cause  
of  MS  (others  congenital,  SLE,  RA)  
•  LeW  atrial  myxoma  mimics  symptoms  
•  Accentuated  S1,  diastolic  rumble,  opening  
snap  
Mitral  Stenosis  (MS)  

LA  enlargement  on  CXR  and  ECG  (wide,  notched  P  wave)  


Mitral  Stenosis  (MS)  

•  Left Atrial Enlargement


–  PA view:
•  Double density
•  Enlargement of LA
appendage
•  Upliftment of left mainstem
bronchus
•  Widening of carinal angle
3.  Which  ECG  finding  disLnguishes  
acute  MI  from  pericardiLs?  

A.  ST  elevaLons  are  concave  


B.  Development  of  Q  waves  
C.  Tall  P  waves  
D.  T  wave  inversions  usually  seen  within  days  
before  ST  segments  become  isoelectric  
3.  Which  ECG  finding  disLnguishes  acute  
MI  from  pericardiLs?  
A.  ST  elevaLons  are  concave  
B.  Development  of  Q  waves  
C.  Tall  P  waves  
D.  T  wave  inversions  usually  seen  within  days  
before  ST  segments  become  isoelectric  
Electocardiographic  Changes  in    
ACUTE  PERICARDITIS  
(4  Stages)  
 
Stage  1                  Widespread  elevaLon  of  the  ST  segments,  
       oWen    with  upward  concavity,  reciprocal                    
       depressions  only  in  aVR  and/or  V ,  as  well  
1

       as  PR  depression.  


Stage  2      ST  segment  return  to  normal  
Stage  3      T  wave  inversion  
Stage  4      Normal  
   
In  contrast,  in  AMI:  
•  ST  ElevaLons  are  convex  
•  Reciprocal  depression  more  prominent  
•  Development  of  Q  waves    
•  Notching  and  loss  or  R-­‐wave  amplitude  
•  T-­‐wave  inversions  are  usually  seen  within  
hours  before  the  ST  segments  have  
become  isoelectric  
4.  DiagnosLc  feature  of  cardiac  
tamponade  
A.  Pericardial  knock  
B.  Pulsus  paradoxus  
C.  Thickened  and  calcified  pericardium  
D.  Increased  myocardial  thickness  
4.  DiagnosLc  feature  of  cardiac  
tamponade  
A.  Pericardial  knock  
B.  Pulsus  paradoxus  
C.  Thickened  and  calcified  pericardium  
D.  Increased  myocardial  thickness  
Cardiac  Tamponade  
•  Beck’s  triad  (hypotension,  soW/muffled  heart  
sounds,  distended  neck  veins)  
•  Pulsus  paradoxus:  greater  than  normal  (10  
mmHg)  inspiratory  decline  in  systolic  arterial  
pressure    
•  Prominent  x  descent  
•  Electrical  alternans  
•  Pericardial  effusion  
•  EqualizaLon  of  diastolic  pressures  
Table  239-­‐2  Features  That  DisNnguish  Cardiac  Tamponade  from  ConstricNve  PericardiNs    
and  Similar  Clinical  Disorders  
CharacterisLc   Tamponade   ConstricNve   RestricNve   RVMI  
PericardiNs   Cardiomyopathy  
Clinical                    
Pulsus  paradoxus   Common   Usually  absent   Rare   Rare  
Jugular  veins                  
   Prominent  y  descent     Absent   Usually  present   Rare   Rare  
   Prominent  x  descent     Present   Usually  present   Present   Rare  
Kussmaul's  sign   Absent   Present   Present   Present  
Third  heart  sound   Absent   Absent   Rare   May  be  present  
Pericardial  knock   Absent   OWen  present   Absent   Absent  
Electrocardiogram                    
Low  ECG  voltage   May  be  present   May  be  present   May  be  present   Absent  
Electrical  alternans   May  be  present   Absent   Absent   Absent  
Echocardiography                    
Thickened  pericardium   Absent   Present   Absent   Absent  
Pericardial  effusion   Present   Absent   Absent   Absent  
RV  size   Usually  small   Usually  normal   Usually  normal   Enlarged  
Myocardial  thickness   Normal   Normal   Usually  increased   Normal  
Right  atrial  collapse  and   Present   Absent   Absent   Absent  
RVDC  
Increased  early  filling,  ↑   Absent   Present   Present   May  be  present  
mitral  flow  velocity  
Exaggerated  respiratory   Present   Present   Absent   Absent  
variaLon  in  flow  velocity  
5.  IniLal  anL-­‐hypertensive  therapy  
with  thiazide  diureLc  should  be  given  
to  which  paLent?  
A.  48/M  CHF  FuncLonal  Class  III,  BP  160/90  
B.  50/M  BP  140/90  with  gouty  arthriLs  
C.  23/F  with  CKD  stage  5  from  chronic  
glomerulonephriLs  BP  150/90  
D.  55/F  with  dyslipidemia  &  bronchial  asthma,  
BP  150/90  
5.  IniLal  anL-­‐hypertensive  therapy  
with  thiazide  diureLc  should  be  given  
to  which  paLent:  
A.  48/M  CHF  FuncLonal  Class  III,  BP  160/90  
B.  50/M  BP  140/90  with  gouty  arthriLs  
C.  23/F  with  CKD  stage  5  from  chronic  
glomerulonephriLs,  BP  150/90  
D.  55/F  with  dyslipidemia  &  bronchial  asthma,  
BP  150/90  
Drugs  for  Hypertension  
Drug  Class   Examples   IndicaNons   ContraindicaNons  
Thiazides   Hydrochlorothiazide   IniLal  therapy   Diabetes,  Gout,  
Chlorthalidone   Hyperuricemia,  
Hypokalemia  
Loop  diureNcs   Furosemide   CHF,  renal  failure   Diabetes  
Ethacrynic  acid   Hypokalemia  
Aldosterone   Spironolactone   CHF,  primary   Renal  failure  
antagonists   Eplerenone   aldosteronism   Hyperkalemia  
Beta  blockers        
Cardioselec-ve   Metoprolol   Angina   Asthma    
  Atenolol   CHF  (systolic)   COPD  
Nonselec-ve   Propranolol   Post-­‐MI   2nd  or  3rd  degree  
    Sinus  tachycardia   AV  block  
Combined   Carvedilol   Ventricular   Sick  sinus  
alpha/beta   Labetalol   tachyarrhythmia   syndrome  
antagonist    
   
Drugs  for  Hypertension  
Drug  Class   Examples   IndicaNons   ContraindicaNons  
ACE  inhibitors   Captopril   Post-­‐MI/CHD   Acute  renal  failure,  
Enalapril   CHF  with  low  EF   bilateral  renal  artery  
Lisinopril   Nephropathy   stenosis,  
hyperkalemia,  
Pregnancy  
Angiotensin   Losartan   CHF  with  low  EF   Same  as  ACEI  
receptor   Valsartan   Nephropathy  
blockers   Olmesartan   ACE-­‐induced  cough  
Calcium        
antagonists        
Dihydropyridine   Nifedipine      
  Felodipiine      
       
Non-­‐ Verapamil   Post-­‐MI,    angina,   2nd  or  3rd  degree  AV  
dihydropyridine   DilLazem   Supraventricular   block  
tachycardias  
Drugs  for  Hypertension  
Drug  Class   Examples   IndicaNons   ContraindicaNons
/  CauNon  
Alpha      
antagonists      
Selec-ve   Prazosin,  Terazosin   ProstaLsm  
     
Nonselec-ve   Phenoxybenzamine   Pheochromocytoma  
SympatholyNcs   Clonidine  
Methyldopa  
Reserpine  
Direct   Hydralazine   Severe  Coronary  
vasodilators   Minoxidil   Artery  Disease  
Renin  inhibitors   Aliskiren   DiabeLc   Pregnancy  
nephropathy  

HPIM  18th  ed  


tiate pharmacologic treatment to lower BP at SBP !140 mm Hg or DBP !90
mm Hg and treat to goal SBP <140 mm Hg and goal DBP <90 mm Hg. (Expert
2014 Guideline for Management of High Blood Pressure Opinion – Grade E) Special Communication Clinical Review & Education
Recommendation 5

Box. Recommendations for Management of Hypertension


JNC  VIII  In the population aged !18 years with diabetes, initiate pharmacologic treat-
ment to lower BP
to increase. atof
In 2 SBPthe trialsmm
!140 Hgprovide
that or DBP !90 mm Hg
evidence and treat toan
supporting
SBP <140 mm Hg and goal DBP <90 mm Hg. (Expert Opinion – Grade E)
goal lower than 150 mm Hg, the average treated SBP was 143 to 144
a goal
SBP

Recommendation 1
mm Hg.2,3 Many6participants in those studies achieved an SBP lower
Recommendation
Inthan
the general
140 mm nonblack
Hg withpopulation,
treatmentincluding
that wasthose with diabetes,
generally initial
well tolerated.
In the general population aged !60 years, initiate pharmacologic treat-
antihypertensive
Two other trials treatment
9,10 should include a thiazide-type diuretic,
suggest there was no benefit for an SBP goal lower cal-
ment to lower blood pressure (BP) at systolic blood pressure (SBP) !150
cium channel blocker (CCB), angiotensin-converting enzyme inhibitor
mm Hg or diastolic blood pressure (DBP) !90 mm Hg and treat to a goal than 140 mm Hg, but the confidence intervals around the effect sizes
(ACEI), or angiotensin receptor blocker (ARB). (Moderate Recommenda-
SBP <150 mm Hg and goal DBP <90 mm Hg. (Strong Recommendation –
were
tion wideB)and did not exclude the possibility of a clinically impor-
– Grade
Grade A)
tant benefit. Therefore, the panel included a corollary recommen-
Recommendation 7
Corollary Recommendation dation based on expert opinion that treatment for hypertension does
In the general black population, including those with diabetes, initial anti-
In the general population aged !60 years, if pharmacologic treatment for not need totreatment
be adjusted if treatment results in SBP lower
hypertensive should include a thiazide-type diuretic or than 140
CCB. (For
high BP results in lower achieved SBP (eg, <140 mm Hg) and treatment is
mm Hg
general andpopulation:
black is not associated
Moderate with adverse effects–on
Recommendation health
Grade or black
B; for qual-
well tolerated and without adverse effects on health or quality of life, treat-
patients with
ity of life. diabetes: Weak Recommendation – Grade C)
ment does not need to be adjusted. (Expert Opinion – Grade E)
While all panel
Recommendation 8 members agreed that the evidence supporting
Recommendation 2
Inrecommendation
the population aged 1 is!18
veryyears
strong,
withthe panel
CKD, was(or
initial unable
add-on)to reach una-
antihyper-
In the general population <60 years, initiate pharmacologic treatment to
tensive
nimity on the recommendation of a goal SBP of lower than 150out-
treatment should include an ACEI or ARB to improve kidney mm
lower BP at DBP !90 mm Hg and treat to a goal DBP <90 mm Hg. (For ages
comes. This applies to all CKD patients with hypertension regardless of race
30-59 years, Strong Recommendation – Grade A; For ages 18-29 years, Hg. Some members recommended continuing the JNC 7 SBP goal
or diabetes status. (Moderate Recommendation – Grade B)
Expert Opinion – Grade E) of lower than 140 mm Hg for individuals older than 60 years based
Recommendation
on expert opinion. 9 12 These members concluded that the evidence
Recommendation 3
The main objective of hypertension treatment is to attain and maintain goal
In the general population <60 years, initiate pharmacologic treatment to was insufficient to raise the SBP target from lower than 140 to lower
BP. If goal BP is not reached within a month of treatment, increase the dose
lower BP at SBP !140 mm Hg and treat to a goal SBP <140 mm Hg. (Expert ofthan 150 mm
the initial drugHg in high-risk
or add a secondgroups,
drug fromsuchoneas ofblack persons,
the classes those
in recom-
Opinion – Grade E) with CVD6including
mendation stroke,
(thiazide-type and those
diuretic, withormultiple
CCB, ACEI, ARB). The risk factors.
clinician The
should
Recommendation 4 continue to assess BP and adjust the treatment regimen
panel agreed that more research is needed to identify optimal goals until goal BP is
reached. If goal BP cannot be reached with 2 drugs, add and titrate a third
In the population aged !18 years with chronic kidney disease (CKD), ini- of SBP for patients with high BP.
drug from the list provided. Do not use an ACEI and an ARB together in the
tiate pharmacologic treatment to lower BP at SBP !140 mm Hg or DBP !90
same patient. If goal BP cannot be reached using only the drugs in recom-
mm Hg and treat to goal SBP <140 mm Hg and goal DBP <90 mm Hg. (Expert
Opinion – Grade E) Recommendation
mendation 6 because of2a contraindication or the need to use more than 3
drugs to reach
In the generalgoal BP, antihypertensive
population younger thandrugs
60from other
years, classes
initiate can be
pharma-
Recommendation 5 used. Referral to a hypertension specialist may be indicated for patients in
cologic treatment to lower BP at DBP of 90 mm Hg or higher and
In the population aged !18 years with diabetes, initiate pharmacologic treat- whom goal BP cannot be attained using the above strategy or for the man-
ment to lower BP at SBP !140 mm Hg or DBP !90 mm Hg and treat to a goal
treat toof
agement a goal DBP of lower
complicated than
patients for 90 mmadditional
whom Hg. clinical consulta-
SBP <140 mm Hg and goal DBP <90 mm Hg. (Expert Opinion – Grade E) Forisages
tion 30 through
needed. 59 years,
(Expert Opinion Strong
– Grade E)Recommendation – Grade A
For ages 18 through 29 years, Expert Opinion – Grade E
Recommendation 6
Mechanism of Action
Despite these and other differences, the various beta blockers now
BETA  BLOCKERS  
available are approximately as effective as antihypertensive agents. A

Beta-adrenoceptor blocking drugs

Non-selective Selective With alpha-


blocking
activity
– + – +
ISA ISA ISA ISA

Nadolol Pindolol Atenolol Acebutolol Labetalol


Propranolol Carteolol Esmolol (Practolol) Bucindolol
Timolol Penbutolol Metoprolol Celiprolol Carvedilol
Sotalol Alprenolol Bisoprolol
Tertalolol Oxprenolol Betaxolol
Bevantolol
6.  Treatment  of  choice  for  acute  
pericardiLs  occuring  post-­‐STEMI  

A.  Ibuprofen  200mg  3  Lmes  daily  


B.  Aspirin  650mg  4  Lmes  daily  
C.  Prednisone  1mg/kg  daily  
D.  Warfarin  2.5  to  5mg  daily  
6.  Treatment  of  choice  for  acute  
pericardiLs  occuring  post-­‐STEMI  

A.  Ibuprofen  200mg  3  Lmes  daily  


B.  Aspirin  650mg  4  Nmes  daily  
C.  Prednisone  1mg/kg  daily  
D.  Warfarin  2.5  to  5mg  daily  
7.  A  64/M  came  to  the  ER  for  chest  heaviness  starLng  
3  hours  prior,  unrelieved  by  nitrates  given  to  him  at  
the  previous  hospital.    On  PE,  BP  80/50    HR  64    RR  20  
neck  veins  were  flat,  breath  sounds  clear  while  heart  
sounds  were  disLnct.    On  12L  ECG,  ST  elevaLon  was  
seen  in  leads  II,  III  and  AVF.  What  is  the  next  step  in  
stabilizing  this  paLent?    

A.  Give  morphine    


B.  Give  intravenous  fluids  
C.  Start  dopamine  drip  
D.  Start  dobutamine  drip  
7.  A  64/M  came  to  the  ER  for  chest  heaviness  starLng  
3  hours  prior,  unrelieved  by  nitrates  given  to  him  at  
the  previous  hospital.    On  PE,  BP  80/50    HR  64    RR  20  
neck  veins  were  flat,  breath  sounds  clear  while  heart  
sounds  were  disLnct.    On  12L  ECG,  ST  elevaLon  was  
seen  in  leads  II,  III  and  AVF.  What  is  the  next  step  in  
stabilizing  this  paLent?    
A.  Give  morphine    
B.  Give  intravenous  fluids  
C.  Start  dopamine  drip  
D.  Start  dobutamine  drip  
8.  FibrinolyLc  therapy  can  be  given  to  
which  of  the  following  paLents  as  
treatment  for  STEMI:  
A.  64/M  presenLng  with  severe,  tearing  chest  
pain  
B.  49/M  diabeLc  with  BP  190/110  
C.  45/F  with  lupus  nephriLs  and  acLve  menses  
D.  40/M  who  had  ischemic  stroke  6  months  ago  
8.  FibrinolyLc  therapy  can  be  given  to  
which  of  the  following  paLents  as  
treatment  for  STEMI:  
A.  64/M  presenLng  with  severe,  tearing  chest  
pain  
B.  49/M  diabeLc  with  BP  190/110  
C.  45/F  with  lupus  nephriNs  and  acNve  menses  
D.  40/M  who  had  ischemic  stroke  6  months  ago  
Absolute  ContraindicaNons  to  
Fibrinolysis  
•  History  of  cerebrovascular  hemorrhage  at  any  
Lme  
•  Nonhemorrhagic  stroke  or  other  
cerebrovascular  event  within  the  past  year  
•  Marked  hypertension  (SBP>180  mmHg  and/or  
a  DBP>110  mmHg)  at  any  Lme  during  the  
acute  presentaLon  
•  Suspicion  of  aorLc  dissecLon  
•  AcLve  internal  bleeding  (excluding  menses).  
RelaNve  ContraindicaNons  to  
Fibrinolysis  
•  Current  use  of  anLcoagulants  (internaLonal  
normalized  raLo  2)  
•  Recent  (<2  weeks)  invasive  or  surgical  procedure  or  
prolonged  (>10  min)    
•  Cardiopulmonary  resuscitaLon  
•  Known  bleeding  diathesis  
•  Pregnancy,    
•  Hemorrhagic  ophthalmic  condiLon  (e.g.,  hemorrhagic  
diabeLc  reLnopathy)  
•  AcLve  pepLc  ulcer  disease  
•  History  of  severe  hypertension  that  is  currently  
adequately  controlled  
9.  In  assessment  of  blood  pressure,  
using  an  inappropriately  small  BP  cuff  
will  result  in:  
 
A.  OveresLmaLon  of  the  true  blood  pressure  
B.  UnderesLmaLon  of  the  true  blood  
pressure  
C.  No  change  in  the  true  blood  pressure  
D.  OveresLmaLon  only  of  the  systolic  blood  
pressure  
9.  In  assessment  of  blood  pressure,  
using  an  inappropriately  small  BP  cuff  
will  result  in:  
 
A.  OveresNmaNon  of  the  true  blood  pressure  
B.  UnderesLmaLon  of  the  true  blood  
pressure  
C.  No  change  in  the  true  blood  pressure  
D.  OveresLmaLon  only  of  the  systolic  blood  
pressure  
10.    A  59  year  old  male  presents  with  chest  
pain.    On  auscultaLon,  you  were  able  to  
appreciate  a  midsystolic  murmur  radiaLng  
to  the  caroLds.    You  also  noted  a  weak  and  
delayed  pulse.    Your  likely  diagnosis  is:  
 
A. AorLc  stenosis  
B. AorLc  regurgitaLon  
C. Acute  myocardial  infarcLon  
D. Pulmonary  embolism  
 
10.    A  59  year  old  male  presents  with  chest  
pain.    On  auscultaLon,  you  were  able  to  
appreciate  a  midsystolic  murmur  radiaLng  
to  the  caroLds.    You  also  noted  a  weak  and  
delayed  pulse.    Your  likely  diagnosis  is:  
 
A. AorNc  stenosis  
B. AorLc  regurgitaLon  
C. Acute  myocardial  infarcLon  
D. Pulmonary  embolism  
 
11.  A  75  year  old  male  comes  in  for  angina.    He  
is  a  heavy  smoker  and  alcoholic  beverage  
drinker.    He  was  also  said  to  have  a  “fasy  liver”  
by  ultrasound.    On  further  history,  he  admits  to  
have  been  taking  “the  blue  pill”  (sildenafil)  for  
erecLle  dysfuncLon  (last  intake  was  the  day  
prior).    Which  anL-­‐anginal  drug  is  absolutely  
contraindicated  for  this  paLent:  
 
A. Metoprolol    
B. DilLazem  
C. Nitroglycerin  
D. Aspirin  
11.  A  75  year  old  male  comes  in  for  angina.    He  
is  a  heavy  smoker  and  alcoholic  beverage  
drinker.    He  was  also  said  to  have  a  “fasy  liver”  
by  ultrasound.    On  further  history,  he  admits  to  
have  been  taking  “the  blue  pill”  (sildenafil)  for  
erecLle  dysfuncLon  (last  intake  was  the  day  
prior).    Which  anL-­‐anginal  drug  is  absolutely  
contraindicated  for  this  paLent:  
 
A. Metoprolol    
B. DilLazem  
C. Nitroglycerin  
D. Aspirin  
12.  A  35  year  old  male  with  a  heavy  
smoking  history  develops  claudicaLon  on  
the  right  foot  with  gangrene  on  the  Lps  of  
the  toes.    Popliteal  pulses  on  both  
extremiLes  are  normal.    Your  likely  
diagnosis  is:  
 
A. Atheroembolism  
B. Deep  venous  thrombosis  
C. Fibromuscular  dysplasia  
D. ThromboangiiLs  obliterans  
 
12.  A  35  year  old  male  with  a  heavy  
smoking  history  develops  claudicaLon  on  
the  right  foot  with  gangrene  on  the  Lps  of  
the  toes.    Popliteal  pulses  on  both  
extremiLes  are  normal.    Your  likely  
diagnosis  is:  
 
A. Atheroembolism  
B. Deep  venous  thrombosis  
C. Fibromuscular  dysplasia  
D. ThromboangiiNs  obliterans  
 
Buerger’s  Disease  
QUESTION  13  
A  65  year  old  male  was  hospitalized  for  stroke  8  
months  prior  and  was  subsequently  bedridden  
aWer.  He  was  rushed  to  the  emergency  room  
for  sudden  onset  difficulty  of  breathing.    
 
Upon  arrival  at  the  emergency  room,  there  was  
note  of  blood-­‐Lnged  sputum,  with  the  
following  vital  signs  BP:  110/70  HR:  72  regular,  
RR:  34  and  there  was  clear  breath  sounds.  
Emergency  2D  echo  showed  McConnell’s  sign.    
13.  What  is  the  most  common  
cause  of  death  for  paLents  with  
this  condiLon?  

A.  Pump  failure  from  myocardial  ischemia  


B.  Progressive  right  heart  failure  
C.  Respiratory  failure  from  hypoxemia  
D.  Sudden  cardiac  death  from  cardiac  
dysrhythmia  
13.  What  is  the  most  common  
cause  of  death  for  paLents  with  
this  condiLon?  
A.  Pump  failure  from  myocardial  ischemia  
B.  Progressive  right  heart  failure  
C.  Respiratory  failure  from  hypoxemia  
D.  Sudden  cardiac  death  from  cardiac  
dysrhythmia  
Pulmonary  embolism  (PE)  
•  Dyspnea  is  the  most  common  symptom  
•  Tachypnea  is  the  most  common  sign  
•  Risk  factors:    Bedridden  >  3  days  
                   AcLve  Cancer  
                   Major  surgery  <  12  wks  
•  Progressive  right  heart  failure  is  the  usual  
cause  of  death  from  PE  
SIGNS  
•  McConnell’s  Sign:  hypokinesis  of  the  RV  Free  
Wall  with  NORMAL  moLon  of  the  RV  Apex    
•  Homan’s  sign:  posterior  calf  pain  on  acLve  
dorsiflexion  of  the  foot  against  resistance  (for  
DVT)  
•  Hamman’s  sign:  crunching  /  clicking  noise  
synchronous  with  the  heartbeat  &  best  heard  
in  the  leW  lateral  decubitus  posiLon  
QUESTION  14  
 A  54  year  old  diabeLc  male,  with  a  60  pack  
year  smoking  history  sought  consult  at  the  
OPD  1  week  ago  for  exerLonal  dyspnea  and  
cough  with  whiLsh  sputum  which  started  8  
months  ago.  He  was  rushed  to  the  emergency  
room  for  a  3  day  history  of  worsening  
difficulty  of  breathing,  cough  and  decreased  
sensorium.  Upon  arrival,  he  was  seen  drowsy  
with  absent  breath  sounds  and  bipedal  
edema.  
14.  Which  of  the  following  should  be  
the  next  step  in  managing  this  
paLent?  
A.  Intubate  the  paLent  
B.  Nebulize  with  Salbutamol  
C.  Start  theophylline  drip  
D.  Give  Furosemide  IV  boluses  
14.  Which  of  the  following  should  be  
the  next  step  in  managing  this  
paLent?  
A.  Intubate  the  paNent  
B.  Nebulize  with  Salbutamol  
C.  Start  theophylline  drip  
D.  Give  Furosemide  IV  boluses  
15.  A  diagnosLc  thoracentesis  is  
warranted  for  which  of  the  following  
situaLons?  
A.  Pleural  effusions  are  equally  present  on  
both  lung  fields.  
B.  PaLent  is  febrile  
C.  Free  fluid  separates  the  lung  from  the  
chest  wall  by  >10  mm  
D.  Sputum  AFB  is  posiLve  
15.  A  diagnosLc  thoracentesis  is  
warranted  for  which  of  the  following  
situaLons?  
A.  Pleural  effusions  are  equally  present  on  
both  lung  fields.  
B.  PaLent  is  febrile  
C.  Free  fluid  separates  the  lung  from  the  
chest  wall  by  >10  mm  
D.  Sputum  AFB  is  posiLve  
TransudaLve  vs  ExudaLve?    
Light’s  Criteria  
•  Pleural  fluid  protein/serum  protein  >0.5  
•  Pleural  fluid  LDH/serum  LDH  >0.6  
•  Pleural  fluid  LDH  more  than  two-­‐thirds  normal  
upper  limit  for  serum  
Factors  indicaNng  the  likely  need  for  a  
procedure  more  invasive  than  a  
thoracentesis  (increasing  order  of  
importance)  
•  Loculated  pleural  fluid  
•  Pleural  fluid  pH  <7.20  
•  Pleural  fluid  glucose  <3.3  mmol/L  (<60  mg/dL)  
•  PosiLve  Gram  stain  or  culture  of  the  pleural  
fluid  
•  Presence  of  gross  pus  in  the  pleural  space  
QUESTION  16  
 A  50  year  old  female,  on  chronic  dialysis,  was  
transferred  from  a  nursing  home  for  
decreased  in  sensorium.  On  history,  there  was  
a  1  week  history  of  cough  with  yellowish  
sputum,  associated  with  high  grade  fever  and  
progressive  difficulty  of  breathing.    
 On  examinaLon,  she  was  drowsy,  with  coarse  
crackles  on  the  right  lower  lung  field.  Vital  
signs  were  BP:  70/50  HR  118  and  RR  27  and  
febrile  at  39.2°C.  
16.  Which  of  the  following  would  put  
her  at  risk  of  Pseudomonas  aeruginosa  
infecLon?  

A.  Chronic  dialysis  
B.  Family  member  with  a  Pseudomonas  infecLon  
C.  Hypotension  despite  fluid  resuscitaLon  
D.  Living  in  a  nursing  home  
16.  Which  of  the  following  would  put  
her  at  risk  of  Pseudomonas  aeruginosa  
infecLon?  
A.  Chronic  dialysis  
B.  Family  member  with  a  Pseudomonas  infecLon  
C.  Hypotension  despite  fluid  resuscitaLon  
D.  Living  in  a  nursing  home  
Clinical  CondiLons  Associated  with  and  
Likely  Pathogens  in  Health  Care–
Associated  Pneumonia  
CondiNon   MRSA   Pseudomonas   Acinetobacter   MDR  Enterobac-­‐
teriaciae  
HospitalizaLon  for  48  h   X   X   X   X  
HospitalizaLon  for  2   X   X   X   X  
days  in  prior  3  months  
Nursing  home  or   X   X   X   X  
extended-­‐care-­‐facility  
residence  
AnLbioLc  therapy  in   X   X  
preceding  3  months  
Chronic  dialysis   X  
Home  infusion  therapy   X  
Home  wound  care   X  
Family  member  with   X   X  
MDR  infecLon  
QUESTION  17  
 A  30  year  old  paLent  sought  consult  for  
chronic  cough  for  3  months.  She  was  
previously  diagnosed  with  pulmonary  
tuberculosis  but  only  completed  treatment  for  
3  months.  Laboratory  test  showed  posiLve  
sputum  TB  culture  and  chest  x-­‐ray  revealed  
cavitary  TB.  
17.  Which  of  the  following  makes  
this  paLent  most  likely  to  transmit  
PTB?  
A.  Age  of  the  paLent  
B.  PosiLve  sputum  TB  culture  
C.  Presence  of  cavitary  TB  on  chest  x-­‐ray  
D.  Previous  TB  treatment  
17.  Which  of  the  following  makes  
this  paLent  most  likely  to  transmit  
PTB?  
A.  Age  of  the  paLent  
B.  PosiLve  sputum  TB  culture  
C.  Presence  of  cavitary  TB  on  chest  x-­‐ray  
D.  Previous  TB  treatment  
18.  When  should  treatment  failure  
in  pulmonary  tuberculosis  be  
suspected?  
A.  Chest  x-­‐ray  remained  unchanged  aWer  6  
weeks  
B.  Sputum  AFB  smears  remain  posiLve  
aWer  5  months  
C.  Sputum  cultures  remain  posiLve  aWer  2  
months  
d.  If  paLent  stops  medicaLons  for  1  week  
18.  When  should  treatment  failure  in  
pulmonary  tuberculosis  be  
suspected?  
A.  Chest  x-­‐ray  remained  unchanged  aWer  6  
weeks  
B.  Sputum  AFB  smears  remain  posiNve  
aber  5  months  
C.  Sputum  cultures  remain  posiLve  aWer  2  
months  
D.  If  paLent  stops  medicaLons  for  1  week  
TB  Treatment  Failure/Resistance  
•  If  sputum  AFB  is  posiLve  at  3  months  and  
paLent  is  adherent  
•  If  sputum  cultures  remain  posiLve  at  3  
months  
•  In  some  like  extensive  cavitary  disease  and  
large  numbers  of  organisms,  AFB  smear  
conversion  may  lag  behind  culture  conversion  
19.  What  is  the  mechanism  of  
acLon  of  Rifampicin  ?  
 
A. Inhibits  fasy  acid  synthase  and  mycolic  acid  
synthesis  
B. Inhibits  arabinosyltransferases  involved  in  
cell  wall  synthesis  
C. Inhibits  mycobacterial  DNA-­‐dependent  RNA  
polymerase,  blocking  RNA  synthesis  
D. Mechanism  is  unclear  
19.  What  is  the  mechanism  of  
acLon  of  Rifampicin  ?  
 
A. Inhibits  fasy  acid  synthase  and  mycolic  acid  
synthesis  (INH)  
B. Inhibits  arabinosyltransferases  involved  in  
cell  wall  synthesis  (EMB)  
C. Inhibits  mycobacterial  DNA-­‐dependent  
RNA  polymerase,  blocking  RNA  synthesis  
D. Mechanism  is  unclear  
20.  Which  of  the  following  laboratory/
imaging  finding  suggests  the  presence  
of  chronic  hypoxemia  in  COPD?  

A.  Compensatory  increase  of  bicarbonate  


in  arterial  blood  gas  
B.  FEV1/FVC  <  0.7  on  spirometry  
C.  Low  hematocrit    
D.  Right  ventricular  hypertrophy  
20.  Which  of  the  following  laboratory/
imaging  finding  suggests  the  presence  
of  chronic  hypoxemia  in  COPD?  

A.  Compensatory  increase  of  bicarbonate  


in  arterial  blood  gas  
B.  FEV1/FVC  <  0.7  on  spirometry  
C.  Low  hematocrit    
D.  Right  ventricular  hypertrophy  
21.  A  30  year  old  female  came  in  for  severe  
abdominal  pain  radiaLng  to  the  back.    Amylase  and  
lipase  were  significantly  elevated.    On  the  3rd  
postoperaLve  day,  she  had  sudden  worsening  of  
dyspnea  with  a  respiratory  rate  of  34.    Arterial  blood  
gas  showed  a  PO2  of  40  with  an  FiO2  of  40%.                      
Best  management  for  this  paLent  will  be:  

A. Give  stat  dose  of  Furosemide  IV  bolus  with  BP  


precauLons  
B. Start  IV  broad-­‐spectrum  anLbioLcs  immediately  
C. Increase  FiO2  to  60%  
D. InsLtute  venLlatory  support  with  a  low  Ldal  volume    
21.  A  30  year  old  female  came  in  for  severe  
abdominal  pain  radiaLng  to  the  back.    Amylase  and  
lipase  were  significantly  elevated.    On  the  3rd  
postoperaLve  day,  she  had  sudden  worsening  of  
dyspnea  with  a  respiratory  rate  of  34.    Arterial  blood  
gas  showed  a  PO2  of  40  with  an  FiO2  of  40%.                      
Best  management  for  this  paLent  will  be:  

A. Give  stat  dose  of  Furosemide  IV  bolus  with  BP  


precauLons  
B. Start  IV  broad-­‐spectrum  anLbioLcs  immediately  
C. Increase  FiO2  to  60%  
D. InsNtute  venNlatory  support  with  a  low  Ndal  volume    
22.  How  is  Paragonimus  westermani  
(Lung  Fluke)  transmised?  

A. Respiratory  droplets  
B. IngesLon  of  crayfish  
C. Transfer  from  infected  blood  and  body  
fluids  
D. InhalaLon  of  infecLous  larva  
22.  How  is  Paragonimus  westermani  
(Lung  Fluke)  transmised?  

A. Respiratory  droplets  
B. IngesNon  of  crayfish  
C. Transfer  from  infected  blood  and  body  
fluids  
D. InhalaLon  of  infecLous  larva  
23.  A  22  year  old  college  student  sought  consult  
for  a  7  day  history  of  high  grade  fever.  He  also  
complained  of  headache,  body  malaise,  and  
abdominal  pain.  On  physical  examinaLon,  vital  
signs  were  as  follows:  BP  110/60  HR  70  RR  18  
Temp  39.4°C,  the  liver  was  enlarged,  there  was  
splenomegaly,  and  blanching,  maculopapular  
rash  over  the  anterior  chest.    
What  is  the  primary  consideraLon?  
A.  Dengue  hemorrhagic  fever  
B.  Leptospirosis  
C.  Malaria  
D.  Typhoid  fever  
23.  A  22  year  old  college  student  sought  consult  
for  a  7  day  history  of  high  grade  fever.  He  also  
complained  of  headache,  body  malaise,  and  
abdominal  pain.  On  physical  examinaLon,  vital  
signs  were  as  follows:  BP  110/60  HR  70  RR  18  
Temp  39.4°C,  the  liver  was  enlarged,  there  was  
splenomegaly,  and  blanching,  maculopapular  
rash  over  the  anterior  chest.    
What  is  the  primary  consideraLon?  
A.  Dengue  hemorrhagic  fever  
B.  Leptospirosis  
C.  Malaria  
D.  Typhoid  fever  
24.  Aling  Dahlia  is  a  55  year  old  diabeLc  paLent  
you  haven't  seen  in  years.    She  comes  back  to  
you  with  burning  pain  when  she  urinates  and  
vulvar  itching.    On  closer  inspecLon,  you  note  
clumped  white  vaginal  discharge  on  a  
background  of  erythematous  vaginal  epithelium.  
What  is  your  treatment  of  choice?  

     A.  Metronidazole  2g  orally  single  dose  


     B.  Metronidazole  500mg  BID  for  7  days  
     C.  Fluconazole  150mg  orally  single  dose  
     D.  Azithromycin  2g  orally  single  dose  
24.  Aling  Dahlia  is  a  55  year  old  diabeLc  paLent  
you  haven't  seen  in  years.    She  comes  back  to  
you  with  burning  pain  when  she  urinates  and  
vulvar  itching.    On  closer  inspecLon,  you  note  
clumped  white  vaginal  discharge  on  a  
background  of  erythematous  vaginal  epithelium.  
What  is  your  treatment  of  choice?  

     A.  Metronidazole  2g  orally  single  dose  


     B.  Metronidazole  500mg  BID  for  7  days  
     C.  Fluconazole  150mg  orally  single  dose  
     D.  Azithromycin  2g  orally  single  dose  
25.  Your  cousin  complains  to  you  that  she  has  
been  having  repeated  episodes  of  vomiLng  and  
watery  diarrhea  for  the  past  3  hours.    She  denies  
any  fever,  abdominal  cramps  and  did  not  noLce  
blood  in  her  vomitus  or  stools.  She  tells  you  that  
she  has  just  come  from  a  friend's  wedding  4  
hours  ago  where  she  ate  fried  rice,  potato  salad,  
roast  beef  and  prawns.      
 
Which  of  the  following  organisms  most  probably  
caused  her  complaints  in  addiLon  to  oral  
rehydraLon?  
25.    Which  of  the  following  is  a  
reasonable  treatment  regimen  for  
your  cousin  ?  
     A.  Bismuth  or  Loperamide  
     B.  Bismuth  plus  Clindamycin  
     C.  Ciprofloxacin  plus  Metronidazole  
     D.  Ciprofloxacin  alone  
25.  Which  of  the  following  is  a  
reasonable  treatment  regimen  for  
your  cousin  ?  
     A.  Bismuth  or  Loperamide  
     B.  Bismuth  plus  Clindamycin  
     C.  Ciprofloxacin  plus  Metronidazole  
     D.  Ciprofloxacin  alone  
26.  Most  common  clinical  findings  in  
the  acute  clinical  syndrome  of  HIV  

A. Mucocutaneous  ulceraLon,  Kaposi’s  


sarcoma  
B. Diarrhea,  myalgia,  urethral  discharge  
C. PharyngiLs,  lymphadenopathy,  fever  
D. Weight  loss,  night  sweats,  prostraLon  
26.  Most  common  clinical  findings  in  
the  acute  clinical  syndrome  of  HIV  

A. Mucocutaneous  ulceraLon,  Kaposi’s  


sarcoma  
B. Diarrhea,  myalgia,  urethral  discharge  
C. PharyngiNs,  lymphadenopathy,  fever  
D. Weight  loss,  night  sweats,  prostraLon  
27.  True  regarding  treatment  of  
schistosomiasis    
A.  Drug  of  choice  is  praziquantel  in  2  to  3  doses  
given  in  4  weeks.    
B.  Early  hepatomegaly  and  bladder  lesions  do  
not  resolve  with  chemotherapy.  
C.  Treatment  of  severe  acute  schistosomiasis  or  
Katayama  fever  involves  parasite  eliminaLon  
and  supporLve  treatment  such  as  
glucocorLcoids.  
D.  Treatment  of  schistosmiasis  in  a  paLent  with  
co-­‐infecLon  with  HIV  will  reduce  HIV  viral  
load  and  increase  CD4  counts.  
27.  True  regarding  treatment  of  
schistosomiasis    
A.  Drug  of  choice  is  praziquantel  in  2  to  3  doses  
given  in  4  weeks.    
B.  Early  hepatomegaly  and  bladder  lesions  do  
not  resolve  with  chemotherapy.  
C.  Treatment  of  severe  acute  schistosomiasis  or  
Katayama  fever  involves  parasite  eliminaLon  
and  supporLve  treatment  such  as  
glucocorLcoids.  
D.  Treatment  of  schistosmiasis  in  a  paNent  with  
co-­‐infecNon  with  HIV  will  reduce  HIV  viral  
load  and  increase  CD4  counts.  
28.  Which  of  the  following  vaccines  
can  be  given  safely  to  pregnant  
women?  
 
A.     Measles,  mumps,  rubella  (MMR)  
B.     Varicella    
C.     Tetanus  toxoid  
D.     Herpes  zoster  
28.  Which  of  the  following  vaccines  
can  be  given  safely  to  pregnant  
women?  
 
A.     Measles,  mumps,  rubella  (MMR)  
B.     Varicella    
C.     Tetanus  toxoid  
D.     Herpes  zoster  
29.  Capt.  Reyes  is  a  naval  officer  who  is  
now  going  to  be  assigned  to  Papua  New  
Guinea.    He  consults  you  since  he  is  
concerned  about  malaria  in  the  area  and  
intends  to  bring  his  pregnant  wife  and  12  
y/o  son  with  him.    
 
 Which  of  the  following  should  be  your  
advice  to  them?  
 
A. It  is  very  important  to  apply  anL-­‐mosquito  
repellants  especially  in  mid-­‐aWernoon  when  the  
anopheline  mosquitoes  most  oWen  feed  
B. Travellers  should  take  their  chemoprophylaxis  12-­‐24  
hours  prior  to  departure  and  conLnue  unLl  1  week  
aWer  departure  
C. Pregnant  women  can  be  given  mefloquine  as  
chemoprophylaxis  
D.   Chemoprophylaxis  for  malaria  when  given  
appropriately  assures  protecLon  for  travellers  and  
diagnosis  other  than  malaria  should  be  sought  when  
they  present  with  fever.  
A. It  is  very  important  to  apply  anL-­‐mosquito  
repellants  especially  in  mid-­‐aWernoon  when  the  
anopheline  mosquitoes  most  oWen  feed  
B. Travellers  should  take  their  chemoprophylaxis  12-­‐24  
hours  prior  to  departure  and  conLnue  unLl  1  week  
aWer  departure  
C. Pregnant  women  can  be  given  mefloquine  as  
chemoprophylaxis  
D.   Chemoprophylaxis  for  malaria  when  given  
appropriately  assures  protecLon  for  travellers  and  
diagnosis  other  than  malaria  should  be  sought  when  
they  present  with  fever.  
30.  Which  of  the  following  is  true  
regarding  serologic  tests  for  syphilis?  
A. RPR  remains  the  standard  in  examining  
cerebrospinal  fluid  (CSF)  
B. VDRL  is  the  test  of  choice  for  rapid  serologic  
diagnosis  in  a  clinical  sezng  
C. RPR  Lters  correlate  with  VDRL  Lters  during  
treatment  and  may  be  used  interchangeably  
D. A  non-­‐reacLve  FTA-­‐ABS  test  on  CSF  rules  out  
asymptomaLc  neurosyphilis.  
30.  Which  of  the  following  is  true  
regarding  serologic  tests  for  syphilis?  
A. RPR  remains  the  standard  in  examining  
cerebrospinal  fluid  (CSF)  
B. VDRL  is  the  test  of  choice  for  rapid  serologic  
diagnosis  in  a  clinical  sezng  
C. RPR  Lters  correlate  with  VDRL  Lters  during  
treatment  and  may  be  used  interchangeably  
D. A  non-­‐reacNve  FTA-­‐ABS  test  on  CSF  rules  out  
asymptomaNc  neurosyphilis.  
Syphilis  DiagnosNcs:  Nontreponemal  
•  RPR  test  is  easier  to  perform  and  uses  unheated  
serum;  test  of  choice  for  rapid  serologic  diagnosis  
in  a  clinical  sezng  and  can  be  automated.    
•  VDRL  test  remains  the  standard  for  examining  
CSF.    
•  RPR  and  VDRL  tests  are  recommended  for  
screening  or  for  quanLtaLon  of  serum  anLbody  
•  VDRL  Lters  do  not  correspond  directly  to  RPR  
Lters  
Syphilis  DiagnosNcs:  Treponemal  
•  Treponemal  anLbody–absorbed  (FTA-­‐ABS)  
test  and  the  T.  pallidum  parLcle  aggluLnaLon  
(TPPA)  test  
•  Confirmatory  test  for  syphilis  
•  Has  high  false  negaLve  rate  
•  Cannot  disLnguish  between  current  and  
treated  syphilis  
31.  Mucocutaneous  lesions,  generalized  non-­‐
tender  lymphadenopathy,  skin  erupLons  that  
involves  palms  and  soles  and  rarely,  lues  
maligna  or  severe  necroLc  lesions,  occur  in  
which  stage  of  syphilis  infecLon?  
 
A. Primary  syphilis    
B. Secondary  syphilis    
C. Latent  syphilis    
D. Late  syphilis    
 
31.  Mucocutaneous  lesions,  generalized  non-­‐
tender  lymphadenopathy,  skin  erupLons  that  
involves  palms  and  soles  and  rarely,  lues  
maligna  or  severe  necroLc  lesions,  occur  in  
which  stage  of  syphilis  infecLon?  
 
A. Primary  syphilis    
B. Secondary  syphilis    
C. Latent  syphilis    
D. Late  syphilis    
 
Syphilis  
•  Primary  Lesion  
–  Appears  at  the  site  of  inoculaNon,  usually  persists  for  4–6  weeks,  and  
then  heals  spontaneously.    
•  Secondary  Syphilis  
–  Generalized  parenchymal,  consLtuLonal,  &  mucocutaneous  
manifestaLons  of  Secondary  syphilis  usually  appear  6–8  weeks  aber  
the  chancre  heals.    
•  Latent  Phase  
–  Secondary  lesions  subside  within  2–6  weeks,  and  infecLon  enters  the  
latent  stage  (detectable  ONLY  by  serologic  tesNng)  
•  TerLary  Disease  
–  MOST  common  types  :  
•  Gumma  (a  usually  benign  granulomatous  lesion)  
•  Cardiovascular  syphilis  (usually  involving  the  vasa  vasorum  of  the  ascending  aorta  
and  resulLng  in  aneurysm)  
•  Late  symptomaNc  neurosyphilis  (tabes  dorsalis  and  paresis)  
32.  This  blood  fluke  is  now  
classified  as  a  human  carcinogen  

A. Schistosoma  japonicum    
B. Schistosoma  hematobium    
C. Schistosoma  mekongi    
D. Schistosoma  intercalatum    
32.  This  blood  fluke  is  now  
classified  as  a  human  carcinogen  

A. Schistosoma  japonicum    
B. Schistosoma  hematobium    
C. Schistosoma  mekongi    
D. Schistosoma  intercalatum    
33.    Most  common  clinical  
manifestaLon  of  Neisseria  
meningi-des  in  humans:  

A. AsymptomaLc  colonizaLon  of  the  


nasopharynx  
B. Bacterial  meningiLs    
C. Meningococcal  sepLcemia    
D. Occult  bacteremia  
33.    Most  common  clinical  
manifestaLon  of  Neisseria  
meningi-des  in  humans:  

A. AsymptomaNc  colonizaNon  of  the  


nasopharynx  
B. Bacterial  meningiLs    
C. Meningococcal  sepLcemia    
D. Occult  bacteremia  
34.  A  30  year  old  female  wondered  why  despite  
her  appeLte  she  seemed  to  be  wasLng  away.  She  
has  been  having  vague  abdominal  pain  and  watery  
diarrhea  for  a  month  now,  despite  treatment  with  
Ciprofloxacin.  She  consults  you  regarding  this,  
wondering  if  it  has  anything  to  do  with  her  love  for  
raw  fish.  Upon  doing  a  basery  of  tests,  you  note  an  
albumin  level  of  12  (normal  value:  40).  Fecalysis  
results  showed  peanut-­‐shaped  eggs.  What  is  your  
diagnosis?  
A. TRICHINELLA      
B. CAPILLARIASIS    
C. ENTEROBIASIS    
D. ASCARIASIS    
34.  A  30  year  old  female  wondered  why  despite  
her  appeLte  she  seemed  to  be  wasLng  away.  She  
has  been  having  vague  abdominal  pain  and  watery  
diarrhea  for  a  month  now,  despite  treatment  with  
Ciprofloxacin.  She  consults  you  regarding  this,  
wondering  if  it  has  anything  to  do  with  her  love  for  
raw  fish.  Upon  doing  a  basery  of  tests,  you  note  an  
albumin  level  of  12  (normal  value:  40).  Fecalysis  
results  showed  peanut-­‐shaped  eggs.  What  is  your  
diagnosis?  
A. TRICHINELLA      
B. CAPILLARIASIS    
C. ENTEROBIASIS    
D. ASCARIASIS    
35.  A  53  year  old  male  came  in  at  the  OPD  for  
jaundice.  On  history  he  was  a  heavy  chronic  
alcoholic  beverage  drinker  and  has  a  strong  family  
history  for  hepatocellular  cancer.  On  physical  
examinaLon,  he  has  gynecomasLa  and  a  few  
spider  angiomata.    
 
HepaLLs  profile:  HBsAg,  anL-­‐HCV:  Non-­‐reacLve      
AST  was  twice  elevated  as  the  ALT,  low  albumin,  
prolonged  PT  
 CBC:  HGB  92  HCT  0.23  MCV  102  MCH  35  Platelet  
166    
   
35.  Which  would  be  the  most  likely  
cause  of  this  paLent’s  chronic  liver  
disease?  
 
A.      Alcoholic  liver  disease  
B.      Chronic  hepaLLs  B  infecLon  
C.      Hepatocellular  carcinoma  
D.      Fasy  liver    
35.  Which  would  be  the  most  likely  
cause  of  this  paLent’s  chronic  liver  
disease?  
 
A.      Alcoholic  liver  disease  
B.      Chronic  hepaLLs  B  infecLon  
C.      Hepatocellular  carcinoma  
D.      Fasy  liver    
36.  What  is  your  interpretaLon  of  the  
following  hepaLLs  profile?    
HbS  (-­‐)    AnL-­‐HBs  (+)  AnL-­‐HBc  IgG  (+)  
HBeAg  (-­‐)    AnL-­‐HBe  (+/-­‐)  
 
A.      Chronic  hepaLLs  B,  high  infecLvity  
B.      Late  acute  or  chronic  hepaLLs  B,  low            
infecLvity  
C.      Recovery  from  hepaLLs  B  
D.      VaccinaLon  against  HBV      
36.  What  is  your  interpretaLon  of  the  
following  hepaLLs  profile?    
HbS  (-­‐)    AnL-­‐HBs  (+)  AnL-­‐HBc  IgG  (+)  
HBeAg  (-­‐)    AnL-­‐HBe  (+/-­‐)  
 
A.      Chronic  hepaLLs  B,  high  infecLvity  
B.      Late  acute  or  chronic  hepaLLs  B,  low            
infecLvity  
C.      Recovery  from  hepaNNs  B  
D.      VaccinaLon  against  HBV      
         37.  A  30  year  old  female  sought  consult  for  
melena.  She  is  complaining  of  1  year  history  of  
recurrent  burning  epigastric  pain,  someLmes  
awakening  the  paLent  in  the  middle  of  the  
night  and  was  relieved  by  intake  of  antacids  or  
food.  On  physical  examinaLon,  she  has  
epigastric  tenderness,  orthostaLc  hypotension,  
melena  on  DRE.  
 
What  is  the  procedure  of  choice  to  confirm  the  
cause  of  melena?  
 
37.  What  is  the  procedure  of  choice  
to  confirm  the  cause  of  melena?  

A.      Barium  swallow  
B.      Abdominal  CT  scan  
C.      Upper  GI  endoscopy  
D.      Lower  GI  endoscopy    
 
37.  What  is  the  procedure  of  choice  
to  confirm  the  cause  of  melena?  

A.      Barium  swallow  
B.      Abdominal  CT  scan  
C.      Upper  GI  endoscopy  
D.      Lower  GI  endoscopy    
 
38.  Mr  Tan  is  a  58  year  old  male  company  
execuLve  who  came  in  for  a  prevenLve  check-­‐up.  
History  and  physical  examinaLon  are  unremarkable  
but  you  noted  microcyLc,  hypochromic  anemia  in  
his  CBC.  Urinalysis,  blood  chemistry  and  chest  x-­‐
rays  are  normal.    What  is  your  next  step?  
 
 A.  Reassure  him  that  the  anemia  might  be  from  
stress  
 B.  Order  for  abdominal  CT  scan  
 C.  Order  PSA,  CEA,  AFP  and  LDH    
 D.  Refer  to  GI  for  flexible  sigmoidoscopy    
38.  Mr  Tan  is  a  58  year  old  male  company  
execuLve  who  came  in  for  a  prevenLve  check-­‐up.  
History  and  physical  examinaLon  are  unremarkable  
but  you  noted  microcyLc,  hypochromic  anemia  in  
his  CBC.  Urinalysis,  blood  chemistry  and  chest  x-­‐
rays  are  normal.    What  is  your  next  step?  
 
 A.  Reassure  him  that  the  anemia  might  be  from  
stress  
 B.  Order  for  abdominal  CT  scan  
 C.  Order  PSA,  CEA,  AFP  and  LDH    
 D.  Refer  to  GI  for  flexible  sigmoidoscopy    
39.  A  30  year  old  male  presents  with  
dysphagia  to  solid  and  liquids.    A  barium  
swallow  x-­‐ray  done  showed  tapering  of  the  
esophagus  with  a  beak-­‐like  appearance.    
DiagnosLc  criteria  by  esophageal  manometry  
would  show:  
 
A.     Over-­‐acLvity  of  peristalsis  
B.     Impaired  LES  relaxaLon  
C.     Air  fluid  level  
D.     Dilated  esophagus  
39.  A  30  year  old  male  presents  with  
dysphagia  to  solid  and  liquids.    A  barium  
swallow  x-­‐ray  done  showed  tapering  of  the  
esophagus  with  a  beak-­‐like  appearance.    
DiagnosLc  criteria  by  esophageal  manometry  
would  show:  
 
A.     Over-­‐acLvity  of  peristalsis  
B.     Impaired  LES  relaxaNon  
C.     Air  fluid  level  
D.     Dilated  esophagus  
Esophageal  disorders  
Achalasia   Barium  swallow:  Esophageal  dilataLon,  
tapering  at  the  gastroesophageal  juncLon  
and  an  air-­‐fluid  level  within  the  esophagus  
Manometry:  Impaired  LES  relaxaLon  and  
absent  peristalsis  
Diffuse   “Corkscrew”  esophagus  on  barium  swallow  
esophageal   Manometry:  simultaneous  contracLons,  
spasm   uncoordinated  ("spasLc")  acLvity    
Candida   CharacterisLc  white  plaques  with  friability  
esophagiLs  
40.  A  50  year  old  male,  smoker,  alcoholic  drinker  
comes  in  for  sudden  severe  nonremizng  abdominal  
pain.    Heart  rate  was  noted  to  be  irregularly  
irregular.    There  was  only  very  minimal  tenderness,  
out  of  proporLon  to  the  symptoms.    The  gold  
standard  of  diagnosis  is:  
 
A. Radiograph  showing  bowel-­‐wall  edema  
(“thumbprinLng”)  
B. DemonstraLon  of  pneumatosis  intesLnalis  
C. Oral  and  IV  dynamic  CT  scan  
D. Lapatoromy  
40.  A  50  year  old  male,  smoker,  alcoholic  drinker  
comes  in  for  sudden  severe  nonremizng  abdominal  
pain.    Heart  rate  was  noted  to  be  irregularly  
irregular.    There  was  only  very  minimal  tenderness,  
out  of  proporLon  to  the  symptoms.    The  gold  
standard  of  diagnosis  is:  
 
A. Radiograph  showing  bowel-­‐wall  edema  
(“thumbprinLng”)  
B. DemonstraLon  of  pneumatosis  intesLnalis  
C. Oral  and  IV  dynamic  CT  scan  
D. Lapatoromy  
41.  Which  among  these  diseases  
will  have  a  serum-­‐ascites  albumin  
gradient  of  >1.1  g/dL?  
 
A.      Cirrhosis  
B.      Peritoneal  carcinomatosis  
C.      Tuberculous  peritoniLs  
D.      NephroLc  syndrome  
 
41.  Which  among  these  diseases  
will  have  a  serum-­‐ascites  albumin  
gradient  of  >1.1  g/dL?  
 
A.      Cirrhosis  
B.      Peritoneal  carcinomatosis  
C.      Tuberculous  peritoniLs  
D.      NephroLc  syndrome  
 
SAAG  
– Serum  to  Ascites  Albumin  
Gradient  (SAAG):    
• >1.1g/dL  =  cause  is  MOST  likely  due  
to  PORTAL  HYPERTENSION  (usually  
in  sezng  of  cirrhosis)  
• <  1.1g/dL  =  infecLon    or  malignancy  
42.    A  20  year  old  female  comes  in  for  
abdominal  pain  occurring  3  days  per  
month  in  the  last  three  months.    
Associated  symptoms  include  
improvement  of  pain  with  defecaLon  and  
soW  frequent  stools.    She  most  likely  has:  

A. Irritable  bowel  syndrome  


B. Inflammatory  bowel  disease  
C. GastrointesLnal  tuberculosis  
D. IntesLnal  parasiLsm  
42.    A  20  year  old  female  comes  in  for  
abdominal  pain  occurring  3  days  per  
month  in  the  last  three  months.    
Associated  symptoms  include  
improvement  of  pain  with  defecaLon  and  
soW  frequent  stools.    She  most  likely  has:  

A. Irritable  bowel  syndrome  


B. Inflammatory  bowel  disease  
C. GastrointesLnal  tuberculosis  
D. IntesLnal  parasiLsm  
43.  What  is  the  role  of  lactulose  in  
paLents  with  hepaLc  encephalopathy?  

A.      Evacuate  blood  from  the  


gastrointesLnal  tract  
B.      InacLvate  colonic  bacteria  
C.      Induce  diarrhea    
D.      Promote  colonic  alkalinizaLon    
43.  What  is  the  role  of  lactulose  in  
paLents  with  hepaLc  encephalopathy?  

A.      Evacuate  blood  from  the  


gastrointesLnal  tract  
B.      InacNvate  colonic  bacteria  
C.      Induce  diarrhea    
D.      Promote  colonic  alkalinizaLon    
44.  Which  of  the  following  should  you  
consider  accurate  in  the  management  of  
gallstones  and  its  complicaLons?  
 
 A.      You  can  employ  Ursodeoxycholic  acid  (UDCA)  in  
the  treatment  of  pigment  stones  less  than  5mm  
 B.      ProphylacLc  cholecystectomy  may  be  
considered  for  those  young  paLents  with  low  
surgical  risk  
 C.      Morphine  should  be  employed  in  the  
management  of  severe  pain  in  acute  cholecysLLs  
 D.    Cholecystectomy  is  advised  in  all  paLents  with  
porcelain  gallbladder  
44.  Which  of  the  following  should  you  
consider  accurate  in  the  management  of  
gallstones  and  its  complicaLons?  
 
 A.      You  can  employ  Ursodeoxycholic  acid  (UDCA)  in  
the  treatment  of  pigment  stones  less  than  5mm  
 B.      ProphylacLc  cholecystectomy  may  be  
considered  for  those  young  paLents  with  low  
surgical  risk  
 C.      Morphine  should  be  employed  in  the  
management  of  severe  pain  in  acute  cholecysLLs  
 D.    Cholecystectomy  is  advised  in  all  paNents  with  
porcelain  gallbladder  
45.  Which  of  the  following  is  NOT  a  
criteria  in  the  Child-­‐Pugh  ClassificaLon  
for  staging  of  cirrhosis?  

A. Bilirubin  
B. ParLal  thromboplasLn  Lme    
C. Serum  albumin  
D. Ascites  
 
45.  Which  of  the  following  is  NOT  a  
criteria  in  the  Child-­‐Pugh  ClassificaLon  
for  staging  of  cirrhosis?  

A. Bilirubin  
B. ParNal  thromboplasNn  Nme    
C. Serum  albumin  
D. Ascites  
 
Child-­‐Pugh  ClassificaNon  
•  Serum  albumin  
•  Serum  bilirubin  
•  Prothrombin  Lme  
•  Ascites  
•  HepaLc  encephalopathy  

•  MELD  classificaNon:  InternaLonal  normalized  


raLo  (INR),  serum  bilirubin,  and  serum  creaLnine    
46.  A  75  year  old  male  with  hypertension  and  
dyslipidemia  presents  with  hematochezia  of  
about  ¼  cup.    Which  of  the  following  will  you  
consider  first  as  it  is  the  most  common  cause  
of  hematochezia  among  the  elderly?  
 
A. Bleeding  hemorrhoids  
B. Colon  carcinoma  
C. PepLc  ulcer  disease  
D. DiverLcular  bleeding  
46.  A  75  year  old  male  with  hypertension  and  
dyslipidemia  presents  with  hematochezia  of  
about  ¼  cup.    Which  of  the  following  will  you  
consider  first  as  it  is  the  most  common  cause  
of  hematochezia  among  the  elderly?  
 
A. Bleeding  hemorrhoids  
B. Colon  carcinoma  
C. PepLc  ulcer  disease  
D. DiverNcular  bleeding  
47.    What  is  the  standard  radiologic  
procedure  for  diagnosis  of  
nephrolithiasis?  
 
A.      Abdominal  CT  scan  with  IV  contrast  
B.      KUB-­‐IVP  
C.      Plain  helical  CT  scan  of  the  abdomen  
D.      Ultrasound  of  the  abdomen  
 
47.    What  is  the  standard  radiologic  
procedure  for  diagnosis  of  
nephrolithiasis?  
 
A.      Abdominal  CT  scan  with  IV  contrast  
B.      KUB-­‐IVP  
C.      Plain  helical  CT  scan  of  the  abdomen  
D.      Ultrasound  of  the  abdomen  
 
48.  R.F.  is  a  65  year  old  male  recently  admised  
for  his  first  cycle  of  chemotherapy  for  acute  
leukemia.    Two  days  aWer  his  iniLal  chemo  
dose,  you  note  that  his  urine  output  has  been  
going  down.    
 Physical  examinaLon  revealed  undistended  
bladder  and  posiLve  Chvostek's  sign,  rest  is  
unremarkable.      
 Stat  blood  chemistry  revealed  normal  creaLne  
kinase,  hypocalcemia,  hyperphosphatemia,  
hyperuricemia  and  mild  hyperkalemia.  
Urinalysis  unremarkable  except  for  uric  acid  
crystals  
48.  What  is  your  treatment  of  choice?  

     A.    GlucocorLcoids  
     B.    Intravenous  crystalloids  with  
allopurinol  
     C.  Intravenous  crystalloids  only  
     D.  Intravenous  furosemide  
48.  What  is  your  treatment  of  choice?  

     A.    GlucocorLcoids  
     B.    Intravenous  crystalloids  with  
allopurinol  
     C.  Intravenous  crystalloids  only  
     D.  Intravenous  furosemide  
49.  In  which  of  the  following  individuals  
would  you  recommend  kidney  biopsy?  
A.  64/M  with  diabetes  and  hypertension  for  15  
years,  poor  compliance  to  meds,  complains  to  
you  of  pruritus  and  insomnia,  serum  creaLnine  
350  mmol/L  

B.  21/F  complaining  of  headache  and  hypogastric  


pain.  BP  170/100  HR  90,  urinalysis  reveals  
protein  1+,  RBC  3+,  WBC  1+  ,  RBC  casts  1+  
       
C.  23/M  complaining  of  edema.  He  claims  to  be  
healthy  except  for  a  bout  of  sore  throat  2  weeks  
ago.  BP  160/90,  HR  88,  urinalysis  reveals  protein  
trace,  RBC  2+,  WBC  1+.  
 
 D.  25/M  came  to  the  ER  for  no  urine  output.    You  
note  stable  vital  signs  and  unremarkable  physical  
exam  except  for  mulLple  bruises  on  his  thighs  
and  legs.  Serum  BUN  14.0  mmol/L,  creaLnine  
380  mmol/L,  foley  catheter  has  minimal  tea-­‐
colored  urine  
49.  In  which  of  the  following  individuals  
would  you  recommend  kidney  biopsy?  

A.  64/M  with  diabetes  and  hypertension  for  15  


years,  poor  compliance  to  meds,  complains  to  
you  of  pruritus  and  insomnia,  serum  creaLnine  
350  mmol/L  

B.  21/F  complaining  of  headache  and  hypogastric  


pain.  BP  170/100  HR  90,  urinalysis  reveals  
protein  1+,  RBC  3+,  WBC  1+  ,  RBC  casts  1+  
       
C.  23/M  complaining  of  edema.  He  claims  to  be  
healthy  except  for  a  bout  of  sore  throat  2  weeks  
ago.  BP  160/90,  HR  88,  urinalysis  reveals  protein  
trace,  RBC  2+,  WBC  1+.  
 
 D.  25/M  came  to  the  ER  for  no  urine  output.    You  
note  stable  vital  signs  and  unremarkable  physical  
exam  except  for  mulLple  bruises  on  his  thighs  
and  legs.  Serum  BUN  14.0  mmol/L,  creaLnine  
380  mmol/L,  foley  catheter  has  minimal  tea-­‐
colored  urine  
50.  Which  of  the  following  suggest  
a  chronic  eLology  for  the  kidney  
disease?  

A.  Bilateral  size  of  kidney  on  ultrasound  10  cm  


B.  ElevaLon  of  serum  creaLnine  >6  weeks  
C.  OsteiLs  fibrosa  cysLca  
D.  Presence  of  oliguria  
 
50.  Which  of  the  following  suggest  
a  chronic  eLology  for  the  kidney  
disease?  

A.  Bilateral  size  of  kidney  on  ultrasound  10  cm  


B.  ElevaLon  of  serum  creaLnine  >6  weeks  
C.  OsteiNs  fibrosa  cysNca  
D.  Presence  of  oliguria  
 
51.  A  45  year  old  female  has  been  
complaining  of  dysuria  and  urinary  
frequency  which  started  1  week  prior  to  
admission.  She  now  presented  to  the  
emergency  room  because  of  fever,  chills,  
nausea  and  persistent  vomiLng  and  CVA  
tenderness  on  the  right.  
 
 
 
51.  When  should  you  contemplate  on  
further  imaging  studies  or  urologic  
consult  for  this  paLent?  
 
A.      If  we  find  WBC  casts  on  urinalysis  
B.      When  fever  persists  aWer  therapy  
C.      When  urine  culture  does  not  reveal  any  
organism  
D.      Imaging  studies  and  urologic  consult  are  
always  needed  to  establish  diagnosis  in  this  
case    
51.  When  should  you  contemplate  on  
further  imaging  studies  or  urologic  
consult  for  this  paLent?  
 
A.      If  we  find  WBC  casts  on  urinalysis  
B.      When  fever  persists  aber  therapy  
C.      When  urine  culture  does  not  reveal  any  
organism  
D.      Imaging  studies  and  urologic  consult  are  
always  needed  to  establish  diagnosis  in  this  
case    
52.  Treatment  of  choice  for  acute  
kidney  injury  from  scleroderma  or  
"scleroderma  renal  crisis"  

A.      GlucocorLcoids    
 B.      ACE  inhibitors  
 C.      Sildenafil    
 D.      Hemodialysis    
52.  Treatment  of  choice  for  acute  
kidney  injury  from  scleroderma  or  
"scleroderma  renal  crisis"  

A.      GlucocorLcoids    
 B.      ACE  inhibitors  
 C.      Sildenafil    
 D.      Hemodialysis    
53.  A  57  year  old  diabeLc,  weighing  60kg,  came  to  
the  emergency  room  for  difficulty  of  breathing.  
Blood  chemistry  results  (in  mmol/L):  
 
BUN  15.56    CreaLnine  266        Na  140      
K  4.9                  Ca  1.45        Alb  1  4  
Which  of  the  following  would  you  expect  to  see  in  
this  paLent’s  ECG?  
 
A.      Peaked  T  waves  
B.      Prominent  U  waves  
C.      Prolonged  QT  interval  
D.      Shortened  QT  interval  
53.  A  57  year  old  diabeLc,  weighing  60kg,  came  to  
the  emergency  room  for  difficulty  of  breathing.  
Blood  chemistry  results  (in  mmol/L):  
 
BUN  15.56    CreaLnine  266        Na  140      
K  4.9                  Ca  1.45        Alb  1  4  
Which  of  the  following  would  you  expect  to  see  in  
this  paLent’s  ECG?  
 
A.      Peaked  T  waves  
B.      Prominent  U  waves  
C.      Prolonged  QT  interval  
D.      Shortened  QT  interval  
•  Prolonged  QT:  Hypocalcemia  
•  Short  QT:  Hypercalcemia  
54.  40/M  diagnosed  with  lung  cancer  
underwent  Chest  CT  with  IV  contrast.  
Baseline  creaLnine  prior  to  introducLon  of  IV  
contrast  was  100  mmol/L.  Repeat  serum  
creaLnine  determinaLon  was  215  mmol/
LaWer  24  hours.  What  was  the  best  
intervenLon  that  could  have  prevented  this  
from  occurring?  
  A. HydraLon  with  saline  soluLon  
B. N-­‐acetylcysteine  
C. IV  Sodium  Bicarbonate  
D. Use  of  iodinated  contrast  material  
54.  40/M  diagnosed  with  lung  cancer  
underwent  Chest  CT  with  IV  contrast.  
Baseline  creaLnine  prior  to  introducLon  of  IV  
contrast  was  100  mmol/L.  Repeat  serum  
creaLnine  determinaLon  was  215  mmol/
LaWer  24  hours.  What  was  the  best  
intervenLon  that  could  have  prevented  this  
from  occurring?  
  A. HydraNon  with  saline  soluNon  
B. N-­‐acetylcysteine  
C. IV  Sodium  Bicarbonate  
D. Use  of  iodinated  contrast  material  
55.  CondiLon  characterized  by  
hematuria,  thinning  and  splizng  of  the  
GBMs,  mild  proteinuria,  chronic  
glomerulosclerosis  leading  to  renal  
failure  and  sensorineural  deafness  
A. AnL-­‐GBM  disease    
B. Alport’s  Syndrome    
C. Thin  Basement  Membrane  disease    
D. Nail-­‐Patella  Syndrome    
 
55.  CondiLon  characterized  by  
hematuria,  thinning  and  splizng  of  the  
GBMs,  mild  proteinuria,  chronic  
glomerulosclerosis  leading  to  renal  
failure  and  sensorineural  deafness  
A. AnL-­‐GBM  disease    
B. Alport’s  Syndrome    
C. Thin  Basement  Membrane  disease    
D. Nail-­‐Patella  Syndrome    
 
56.    Most  common  acute  complicaLon  
of  hemodialysis  parLcularly  among  
diabeLc  paLents  
A.     InfecLon    
B.     Muscle  cramps    
C.     Bleeding    
D.     Hypotension    
56.    Most  common  acute  complicaLon  
of  hemodialysis  parLcularly  among  
diabeLc  paLents  
A.     InfecLon    
B.     Muscle  cramps    
C.     Bleeding    
D.     Hypotension    
57.    Most  common  type  of  renal  
stones  
A.     Calcium  stones  
B.     Uric  acid  stones  
C.     CysLne  stones  
D.     Struvite  stones  
 
57.    Most  common  type  of  renal  
stones  
A.     Calcium  stones  
B.     Uric  acid  stones  
C.     CysLne  stones  
D.     Struvite  stones  
 
     58.    A  22  year  old  female  sought  consult  at  the  
out-­‐paLent  department  for  an  enlarging  anterior  
neck  masses  noted  3  years  ago.  This  was  
accompanied  by  heat  intolerance,  palpitaLon,  
tremors.    
 
 On  physical  examinaLon,  there  was  (+)  
exophthalmos  ,  periorbital  edema,  hyperreflexia,  
enlarged  anterior  neck  mass  that  moves  with  
degluLLon,  (-­‐)  cervical  lymphadenopathy  and  
tachycardic  with  an  irregularly  irregular  rhythm.  
58.  What  is  the  expected  finding  for  
this  paLent  on  nuclear  imaging?  
A.      Focal  areas  of  increased  uptake  
B.  Homogenous  gland  with  decreased            
           uptake  
C.      Increased  tracer  uptake  distributed  
homogenously  
D.      Focal  areas  of  decreased  uptake  
 
58.  What  is  the  expected  finding  for  
this  paLent  on  nuclear  imaging?  

A.      Focal  areas  of  increased  uptake  


B.  Homogenous  gland  with  decreased            
           uptake  
C.      Increased  tracer  uptake  distributed  
homogenously  
D.      Focal  areas  of  decreased  uptake  
 
59.      Which  of  the  following  is  correct  
regarding  radioacLve  iodine  (RAI)  
therapy?  
         A.      Propylthiouracil  should  be  stopped  2  days  
before  RAI  treatment  
 B.      LactaLng  women  may  receive  RAI  
treatment  with  cauLon  
 C.      RAI  may  be  given  to  selected  paLents  with  
thyroid  storm  
 D.    AnLthyroid  drugs  should  be  given  to  
elderly  and  those  with  cardiac  diseases  before  
RAI  treatment  
59.      Which  of  the  following  is  correct  
regarding  radioacLve  iodine  (RAI)  
therapy?  
         A.      Propylthiouracil  should  be  stopped  2  days  
before  RAI  treatment  
 B.      LactaLng  women  may  receive  RAI  
treatment  with  cauLon  
 C.      RAI  may  be  given  to  selected  paLents  with  
thyroid  storm  
 D.    AnNthyroid  drugs  should  be  given  to  
elderly  and  those  with  cardiac  diseases  
before  RAI  treatment  
60.  Which  of  the  following  clinical  
features  seen  in  Cushing’s  syndrome  
are  considered  more  specific  and  more  
useful  in  its  diagnosis?  
 
A.  Facial  plethora  
B.  Thin,  silvery  striae  
C.  Distal  myopathy  
D.  Easy  bruisability  
60.  Which  of  the  following  clinical  
features  seen  in  Cushing’s  syndrome  
are  considered  more  specific  and  more  
useful  in  its  diagnosis?  
 
A.  Facial  plethora  
B.  Thin,  silvery  striae  
C.  Distal  myopathy  
D.  Easy  bruisability  
Clinical  Features  More  Specific  to  
Cushing’s  Syndrome  
•  Fragility  of  the  skin,  with  easy  bruising  and  broad  
(>1  cm)  
•  Purplish  striae    
•  Proximal  myopathy  

•  If  ectopic  ACTH  produc-on,  hyperpigmentaLon  


of  the  knuckles,  scars,  or  skin  areas  exposed  to  
increased  fricLon  is  seen  
61.  Which  of  the  following  is  
appropriate  precauLon  regarding  
exercise  in  paLents  with  Type  1  
diabetes?  
A.      Perform  formal  exercise  tesLng  in  paLents  
with  autonomic  neuropathy  
B.      Cancel  the  acLvity  if  blood  glucose  is  less  
than  100  mg/dl  prior  to  exercise  
C.      Avoid  vigorous  exercise  if  there  is  untreated  
nonproliferaLve  DM  reLnopathy  
D.      Inject  insulin  into  exercising  muscle  if  blood  
glucose  is  more  than  300  mg/dl  
61.  Which  of  the  following  is  
appropriate  precauLon  regarding  
exercise  in  paLents  with  Type  1  
diabetes?  
A.      Perform  formal  exercise  tesNng  in  paNents  
with  autonomic  neuropathy  
B.      Cancel  the  acLvity  if  blood  glucose  is  less  
than  100  mg/dl  prior  to  exercise  
C.      Avoid  vigorous  exercise  if  there  is  untreated  
nonproliferaLve  DM  reLnopathy  
D.      Inject  insulin  into  exercising  muscle  if  blood  
glucose  is  more  than  300  mg/dl  
62.      Which  of  the  following  is  appropriate  
in  the  comprehensive  management  and  
follow-­‐up  of  a  paLent  with  diabetes?  

A.    Screening  for  diabeLc  nephropathy  for  


Type  1  diabetes  at  the  Lme  of  diagnosis  
B.    Annual  monitoring  of  blood  pressure  
C.    Lipid  profile  and  creaLnine  quarterly  
D.    Eye  and  foot  examinaLon  by  doctors    
annually  
 
62.      Which  of  the  following  is  appropriate  
in  the  comprehensive  management  and  
follow-­‐up  of  a  paLent  with  diabetes?  

A.    Screening  for  diabeLc  nephropathy  for  


Type  1  diabetes  at  the  Lme  of  diagnosis  
B.    Annual  monitoring  of  blood  pressure  
C.    Lipid  profile  and  creaLnine  quarterly  
D.    Eye  and  foot  examinaNon  by  doctors    
annually  
 
63.  What  is  the  earliest  symptom  of  
pituitary  failure  (trophic  hormone  
failure)  among  adults?  

A.     Hypogonadism  
B.     Growth  retardaLon  
C.     HyperprolacLnemia  
D.     Headache  
 
63.  What  is  the  earliest  symptom  of  
pituitary  failure  (trophic  hormone  
failure)  among  adults?  

A.     Hypogonadism  
B.     Growth  retardaLon  
C.     HyperprolacLnemia  
D.     Headache  
 
•  Trophic  hormone  failure  associated  with  
pituitary  compression  or  destrucNon  usually  
occurs  SEQUENTIALLY:  
–  GH  >  FSH  >  LH  >  TSH  >  ACTH    
–  During  childhood:  growth  retardaLon  is  oWen  the  
presenNng  feature  
–  In  adults:  HYPOGONADISM  is  the  earliest  
symptom  
64.  Most  sensiLve  hormonal  test  for  
phaeochromocytoma  and  paragangliomas,  
that  is  also  considered  to  be  less  
suscepLble  to  false-­‐posiLve  elevaLons  
from  stress    
A.     Plasma  metanephrine    
B.     Urine  vanillylmandelic  acid    
C.     Urine  catecholamines    
D.     Plasma  catecholamines    
 
64.  Most  sensiLve  hormonal  test  for  
phaeochromocytoma  and  paragangliomas,  
that  is  also  considered  to  be  less  
suscepLble  to  false-­‐posiLve  elevaLons  
from  stress    
A.     Plasma  metanephrine    
B.     Urine  vanillylmandelic  acid    
C.     Urine  catecholamines    
D.     Plasma  catecholamines    
 
65.  This  agent  for  dyslipidemia  may  
increase  insulin  resistance  and  worsen  
glycemic  control  at  high  doses  

A.     SimvastaLn    
B.     NicoLnic  acid    
C.     Gemfibrozil    
D.     Fenofibrate    
 
65.  This  agent  for  dyslipidemia  may  
increase  insulin  resistance  and  worsen  
glycemic  control  at  high  doses  

A.     SimvastaLn    
B.     NicoNnic  acid    
C.     Gemfibrozil    
D.     Fenofibrate    
 
66.  Which  of  the  following  represents  
definite  risk  from  hormone  
replacement  therapy  for  menopause?  
 A.      Colorectal  cancer  
 B.      Coronary  heart  disease  
 C.      Endometrial  cancer  
 D.      Ovarian  cancer  
66.  Which  of  the  following  represents  
definite  risk  from  hormone  
replacement  therapy  for  menopause?  
 A.      Colorectal  cancer  
 B.      Coronary  heart  disease  
 C.      Endometrial  cancer  
 D.      Ovarian  cancer  
Hormone  replacement  therapy  
Definite  Risks   Endometrial  cancer  
Venous  thromboembolism  
Gallbladder  disease  
Breast  cancer  
Definite  Benefits   Symptoms  of  menopause  
Osteoporosis  
Uncertain  Risks  and   Coronary  heart  disease  
Benefits   Stroke  
Ovarian  cancer  
Colorectal  cancer  
Diabetes  mellitus  
CogniLve  funcLon  
Women’s  Health  IniLaLve,  2001  
QUESTION  67  
 
A  50  year  old  paLent  came  for  consult  
for  progressive  generalized  body  
weakness.  Laboratory  examinaLon  
revealed  anemia  and  hypercalcemia  
and  the  presence  of  a  pulmonary  mass  
on  chest  x-­‐ray.    
67.  Which  of  the  following  will  be  
appropriate  differenLals  for  the  case?  

A.  Squamous  cell  Lung  cancer  


B.  Small  cell  Lung  cancer  
C.  Pulmonary  tuberculosis  
D.  All  of  the  above  
 
67.  Which  of  the  following  will  be  
appropriate  differenLals  for  the  case?  

A.  Squamous  cell  Lung  cancer  


B.  Small  cell  Lung  cancer  
C.  Pulmonary  tuberculosis  
D.  All  of  the  above  
 
Malignancy-­‐Related  Hypercalcemia  
 
Solid  tumor  with  metastases  
•  Breast  
   
Solid  tumor  with  humoral  mediaLon  of  hyperCa2+  
•  Lung  
•  Kidney  
   
Hematologic  malignancy    
•  MulLple  myeloma  
•  Lymphoma,  Leukemia    
68.  AWer  one  week,  paLent  complained  of  
dyspnea.  facial  swelling,  hoarseness,  difficulty  of  
breathing.  On  physical  examinaLon,  there  was  
dilated  neck  vein,  increased  collateral  veins  over  
the  anterior  chest  and  edema  of  the  face  and  
arms.  What  is  the  treatment  of  choice  for  his  
condiLon?  
     A.  Intravenous  diureLcs  
     B.  Intravenous  glucocorLcoids  
     C.  Emergent  radiotherapy  
     D.  Emergent  surgery  and  decompression  
68.  AWer  one  week,  paLent  complained  of  
dyspnea.  facial  swelling,  hoarseness,  difficulty  of  
breathing.  On  physical  examinaLon,  there  was  
dilated  neck  vein,  increased  collateral  veins  over  
the  anterior  chest  and  edema  of  the  face  and  
arms.  What  is  the  treatment  of  choice  for  his  
condiLon?  
     A.  Intravenous  diureLcs  
     B.  Intravenous  glucocorLcoids  
     C.  Emergent  radiotherapy  
     D.  Emergent  surgery  and  decompression  
SVC  Syndrome  
69.  Which  of  the  following  
suggests  a  beser  prognosis  for  
paLents  with  breast  cancer?  
A.  Estrogen/progesterone  receptor  posiLvity  
B.  High  proporLon  of  cells  in  S-­‐phase  
C.  Overexpression  of  HER2-­‐neu  gene  
D.  Presence  of  microvessels  
 
69.  Which  of  the  following  
suggests  a  beser  prognosis  for  
paLents  with  breast  cancer?  
A.  Estrogen/progesterone  receptor  
posiNvity  
B.  High  proporLon  of  cells  in  S-­‐phase  
C.  Overexpression  of  HER2-­‐neu  gene  
D.  Presence  of  microvessels  
70.  Which  drug  is  a  monoclonal  
anLbody  against  Her2/neu  and  is  
being  used  in  the  treatment  of  breast  
and  gastric  carcinoma?  
A.      Cyclophosphamide    
B.      Trastuzumab    
C.      Rituximab    
D.      ImaLnib  mesylate    
 
70.  Which  drug  is  a  monoclonal  
anLbody  against  Her2/neu  and  is  
being  used  in  the  treatment  of  breast  
and  gastric  carcinoma?  
A.      Cyclophosphamide    
B.      Trastuzumab    
C.      Rituximab    
D.      ImaLnib  mesylate    
 
71.  A  45  year  old  female  was  brought  to  the  
emergency  room  because  of  dizziness.  On  
further  inquiry,  there  was  note  of  menorrhagia  
for  the  past  4  months.  On  complete  blood  
count,  hemoglobin  was  70  g/L,  hematocrit  was  
0.25;  the  MCV  and  MCH  were  low,  rest  of  CBC  
were  normal.  What  is  the  most  appropriate  
form  of  treatment  to  correct  her  anemia?  
A.  Blood  transfusion  
B.  Folic  acid  tablets  
C.  Intravenous  iron  
D.  Intravenous  Vitamin  B12    
71.  A  45  year  old  female  was  brought  to  the  
emergency  room  because  of  dizziness.  On  
further  inquiry,  there  was  note  of  menorrhagia  
for  the  past  4  months.  On  complete  blood  
count,  hemoglobin  was  70  g/L,  hematocrit  was  
0.25;  the  MCV  and  MCH  were  low,  rest  of  CBC  
were  normal.  What  is  the  most  appropriate  
form  of  treatment  to  correct  her  anemia?  
A.  Blood  transfusion  
B.  Folic  acid  tablets  
C.  Intravenous  iron  
D.  Intravenous  Vitamin  B12    
72.  Hans  is  a  29  year  old  who  came  to  the  
clinic  because  he  passed  out  blood  instead  
of  urine.    On  further  invesLgaLon,  you  
note  anemia  both  on  physical  examinaLon  
and  complete  blood  count,  increased  
reLculocyte  count  and  LDH  levels,  
hemoglobinuria  on  urinalysis  and  normal  
KUB  ultrasound.  Bone  marrow  aspirate  
microscopy  reveals  erythroid  hyperplasia.    
72.  Which  of  the  following  lab  results  
will  confirm  the  diagnosis  in  this  case?  

A.  Decreased  bone  marrow  iron  stain    


B.  Undetectable  haptoglobulin  
C.  CD  55-­‐  and  CD  59-­‐  cells  
D.  Fetal  hemoglobin  pasern  
72.  Which  of  the  following  lab  results  
will  confirm  the  diagnosis  in  this  case?  

A.  Decreased  bone  marrow  iron  stain    


B.  Undetectable  haptoglobulin  
C.  CD  55-­‐  and  CD  59-­‐  cells  
D.  Fetal  hemoglobin  pasern  
Paroxysmal  Nocturnal  Hemoglobinuria  
 
•  Triad:    
–  Hemolysis  
–  Pancytopenia  
–  Venous  Thrombosis  
•  Deficiency  with  CD59  and  CD55.  
•  Associated  with  mutaLons  in  PIG-­‐A  gene  
73.  A  20  year  old  female  presents  with  pallor,  
jaundice,  and  icteric  sclerae.    
Hemoglobin  55  g/L,  hematocrit  17%,  RBC  2.5  
x  1012,  reLculocytes  increased,  indirect  
hyperbilirubinemia,  posiLve  direct  Coomb’s  
test.  Peripheral  smear  shows  many  
spherocytes,  polychromatophilic  macrocytes  
and  nucleated  red  cells.  
 
 What  is  your  diagnosis?    
73.  What  is  your  diagnosis?  
A. Iron  deficiency  anemia  
B. AplasLc  anemia  
C. MegaloblasLc  anemia  
D. Autoimmune  hemolyLc  anemia  
 
73.  What  is  your  diagnosis?  
A. Iron  deficiency  anemia  
B. AplasLc  anemia  
C. MegaloblasLc  anemia  
D. Autoimmune  hemolyNc  anemia  
 
74.  During  your  duty  in  the  ER,  a  paLent  was  
referred  to  you  for  dyspnea.  She  is  a  40-­‐year  
old  female  who  was  diagnosed  with  mitral  
stenosis  previously  stable  on  low  dose  
Digoxin.  She  has  a  history  of  fever,  malaise  
and  more  frequent  bowel  movement  in  the  
past  2  days.    
 On  PE,  she  was  agitated,  BP  70/40    HR  110    RR  
24    Temp  39;  mucosa  was  dry,  neck  veins  
were  flat  however  breath  sounds  were  clear,  
heart  sounds  were  disLnct  but  irregularly  
irregular,  abdomen  was  soW.    
74.  What  is  your  next  step?    
A.  Hydrate  the  paLent  with  lactated  
Ringer’s.  
B.  Start  low  dose  dopamine  drip.  
C.  Start  dobutamine  drip.  
D.  Do  immediate  DC  cardioversion  at  200J.  
 
74.  What  is  your  next  step?    
A.  Hydrate  the  paNent  with  lactated  
Ringer’s.  
B.  Start  low  dose  dopamine  drip.  
C.  Start  dobutamine  drip.  
D.  Do  immediate  DC  cardioversion  at  200J.  
75.    You  are  making  rounds  with  your  
OB  consultant.    In  front  of  you  is  a  
35week  AOG  pregnant  paLent  that  
has  been  having  elevated  blood  
pressure  since  2  days  ago.  Right  in  
front  of  you,  the  paLent  had  a  seizure.  
Blood  pressure  was  taken  and  BP  =  
200/110mmHg.    
75.  What  do  you  do?  
A.    Give  Captopril  25mg  tab  sublingual  stat  
every  15  minutes  
B.    Give  magnesium  sulfate  intramuscularly  
C.    Hook  the  paLent  to  Nicardipine  drip  
D.    Load  with  Phenytoin  IV  
75.  What  do  you  do?  
A.    Give  Captopril  25mg  tab  sublingual  stat  
every  15  minutes  
B.    Give  magnesium  sulfate  intramuscularly    
C.    Hook  the  paLent  to  Nicardipine  drip  
D.    Load  with  Phenytoin  IV  
76.    60  year  old  male  suddenly  had  
dizziness  and  diaphoresis.  BP  is  
60/40mmHg  with  cold  clammy  extremiLes.  
ECG  shows  the  following.  What  do  you  do?    
 
76.  What  do  you  do?  

A. Give  IV  Streptokinase  


B. Give  Verapamil  5mg  IV  STAT  
C. Provide  synchronized  electrical  
cardioversion  
D. Start  CPR  
 
76.  What  do  you  do?  

A. Give  IV  Streptokinase  


B. Give  Verapamil  5mg  IV  STAT  
C. Provide  synchronized  electrical  
cardioversion  
D. Start  CPR  
 
77.    A  59  year  old  male  was  brought  in  for  
drowsiness.    Serum  calcium  was  significantly  
elevated.    A  malignancy  is  suspected  as  an  
eLology  for  the  hypercalcemia.  Which  of  the  
following  will  not  be  useful  to  decrease  his  
calcium  levels  immediately?  
A. Intravenous  furosemide  
B. Intravenous  saline  
C. Pamidronate  
D. Calcitonin  
77.    A  59  year  old  male  was  brought  in  for  
drowsiness.    Serum  calcium  was  significantly  
elevated.    A  malignancy  is  suspected  as  an  
eLology  for  the  hypercalcemia.  Which  of  the  
following  will  not  be  useful  to  decrease  his  
calcium  levels  immediately?  
 
A. Intravenous  furosemide  
B. Intravenous  saline  
C. Pamidronate  
D. Calcitonin  
Treatment  of  HyperCa  
78.  A  65  year  old  paLent,  known  hypertensive  
but  poorly  compliant  to  medicaLons,  came  in  
for  altered  mental  status  and  severe  headache.    
Blood  pressure  taken  on  admission  was  
260/100mmHg.    Management  would  include:  
A. Give  captopril  25mg  sublingually  for  three  
doses.    If  there  is  poor  response,  start  
nicardipine  drip  
B. Start  nicardipine  drip  immediately  
C. Give  captopril  25mg  every  8  hours  per  orem  
D. Observe  for  1  hour.    If  symptoms  resolve,  
discharge  on  short  acLng  anL-­‐hypertensives  
78.  A  65  year  old  paLent,  known  hypertensive  
but  poorly  compliant  to  medicaLons,  came  in  
for  altered  mental  status  and  severe  headache.    
Blood  pressure  taken  on  admission  was  
260/100mmHg.    Management  would  include:  
A. Give  captopril  25mg  sublingually  for  three  
doses.    If  there  is  poor  response,  start  
nicardipine  drip  
B. Start  nicardipine  drip  immediately  
C. Give  captopril  25mg  every  8  hours  per  orem  
D. Observe  for  1  hour.    If  symptoms  resolve,  
discharge  on  short  acLng  anL-­‐hypertensives  
79.  A  60  year  old  male  sought  consult  for  
right  knee  joint  pain  which  started  1  year  
ago.  Now  he  presents  with  right  knee  
tenderness  and  effusion  since  4  days  ago  
and  he  also  complains  of  fever  38.0oC.    
 
You  decided  to  perform  arthrocentesis.  
Synovial  fluid  analysis  shows  30,000  cells/
uL  with  70%  neutrophils.  PosiLve  for  rod  
shaped  strongly  negaLve  birefringent  
crystals.  
79.  What  is  your  diagnosis?  
A.  CPPD  disease  
B.  Gouty  arthriLs  
C.  OsteoarthriLs  
D.  SepLc  arthriLs  
79.  What  is  your  diagnosis?  
A.  CPPD  disease  
B.  Gouty  arthriNs  
C.  OsteoarthriLs  
D.  SepLc  arthriLs  
Crystals  
•  Gout:  strongly  negaLve  birefringent  needle-­‐
shaped  or  rod-­‐shaped  
•  CPPD  DeposiLon  Disease:  weakly  posiLve  
birefringent  Rhomboid,  Rectangular  or  Rod-­‐like  
•  Calcium  ApaLte  DeposiLon  Disease:  shiny  intra  or  
extracellular  non-­‐birefringent  globules  or  
aggregates  that  stain  purplish  with  Wright’s  stain  
and  bright  red  with  Alzarin  Red.  
•  Calcium  Oxalate  DeposiLon  Disease:  strongly  
birefringent  bipyramidal  
80.  Which  of  the  following  
characterisLc  symptom  will  favor  the  
diagnosis  of  rheumatoid  arthriLs?  
A.  Migratory  polyarthriLs  
B.  Morning  sLffness  of  10  minutes  
C.  Symmetric  involvement  
D.  Subacute  monoarLcular  arthriLs    
80.  Which  of  the  following  
characterisLc  symptom  will  favor  the  
diagnosis  of  rheumatoid  arthriLs?  
A.  Migratory  polyarthriLs  
B.  Morning  sLffness  of  10  minutes  
C.  Symmetric  involvement  
D.  Subacute  monoarLcular  arthriLs    
81.  DiagnosLc  test  specific  for  
rheumatoid  arthriLs  and  useful  in  
predicLng  its  prognosis  
A. Rheumatoid  Factor    
B. ESR    
C. AnL-­‐CCP  anLbodies    
D. CRP    
 
81.  DiagnosLc  test  specific  for  
rheumatoid  arthriLs  and  useful  in  
predicLng  its  prognosis  
A. Rheumatoid  Factor    
B. ESR    
C. AnN-­‐CCP  anNbodies    
D. CRP    
 
82.  Which  of  the  following  is  an  
inhibitor  of  inosine  monophosphate  
and  purine  synthesis  used  in  the  
treatment  of  SLE?  
A.      Cyclophosphamide    
B.      Methotrexate    
C.      Mycophenolate  mofeLl    
D.      Allopurinol    
 
82.  Which  of  the  following  is  an  
inhibitor  of  inosine  monophosphate  
and  purine  synthesis  used  in  the  
treatment  of  SLE?  
A.      Cyclophosphamide    
B.      Methotrexate    
C.      Mycophenolate  mofeNl    
D.      Allopurinol    
 
83.  AutoanLbody  seen  more  
frequently  in  drug-­‐induced  lupus  
than  in  SLE  

A. AnLphospholipid  
B. AnL-­‐RNP  
C. AnL-­‐histone  
D. AnL-­‐Sm  
 
83.  AutoanLbody  seen  more  
frequently  in  drug-­‐induced  lupus  
than  in  SLE  

A. AnLphospholipid  
B. AnL-­‐RNP  
C. AnN-­‐histone  
D. AnL-­‐Sm  
 
84.  Two  weeks  aWer  starLng  allopurinol  in  
your  paLent  with  gout,  the  paLent  noted  
development  of  blisters  over  8%  of  the  body  
with  note  of  oral  ulcers  and  associated  
diarrhea.  What  is  your  impression?  

A.  Toxic  Epidermal  Necrolysis  


B.  Stevens  Johnson  Syndrome  
C.  Staphylococcal  Scalded  Skin  Syndrome  
D.  Bullous  Pemphigoid  
84.  Two  weeks  aWer  starLng  allopurinol  in  
your  paLent  with  gout,  the  paLent  noted  
development  of  blisters  over  8%  of  the  body  
with  note  of  oral  ulcers  and  associated  
diarrhea.  What  is  your  impression?  

A.  Toxic  Epidermal  Necrolysis  


B.  Stevens  Johnson  Syndrome  
C.  Staphylococcal  Scalded  Skin  Syndrome  
D.  Bullous  Pemphigoid  
Spectrum  
•  Stevens-­‐Johnson  syndrome:  <10%  detached  
•  SJS  /  TEN  Overlap:  10–30%  detached  
•  Toxic  epidermal  necrolysis  overlap:  >30%  
detached  
85.  Nina  came  to  your  clinic  due  to  a  possible  
celluliLs.    She  menLoned  that  she  
experienced  wheals,  chest  pain  and  difficulty  
of  breathing  shortly  aWer  taking  Amoxicillin.  
Which  of  the  following  anLbioLcs  may  sLll  be  
given  safely?  
A.  Piperacillin-­‐tazobactam  
B.  Cefuroxime  
C.  Clindamycin  
D.  Co-­‐amoxyclav  
85.  Nina  came  to  your  clinic  due  to  a  possible  
celluliLs.    She  menLoned  that  she  
experienced  wheals,  chest  pain  and  difficulty  
of  breathing  shortly  aWer  taking  Amoxicillin.  
Which  of  the  following  anLbioLcs  may  sLll  be  
given  safely?  
A.  Piperacillin-­‐tazobactam  
B.  Cefuroxime  
C.  Clindamycin  
D.  Co-­‐amoxyclav  
86.    Most  frequently  associated  
with  the  transfusion  of  cellular  
blood  components  
A.      Febrile  nonhemolyLc  transfusion  
reacLon  
B.      GraW  versus  host  disease  
C.      Transfusion  –  related  acute  lung  injury  
D.    HepaLLs  C  infecLon  
86.    Most  frequently  associated  
with  the  transfusion  of  cellular  
blood  components  
A.      Febrile  nonhemolyNc  transfusion  
reacNon  
B.      GraW  versus  host  disease  
C.      Transfusion  –  related  acute  lung  injury  
D.    HepaLLs  C  infecLon  
87.  Who  of  the  following  paLents  
will  you  diagnose  with  Anaphylaxis?  
A. 45/F  with  maculopapular  rashes  3  weeks  
aWer  starLng  Allopurinol  
B. 50/M  feels  light-­‐headed  with  rashes  aWer  
taking  Co-­‐amoxiclav.  BP  is  60/40  
C. 18/M  with  hives  within  minutes  aWer  eaLng  
crabs  
D. 28/F  OR  nurse  with  rashes  on  her  hands  
usually  noted  aWer  her  duty  
87.  Who  of  the  following  paLents  
will  you  diagnose  with  Anaphylaxis?  
A. 45/F  with  maculopapular  rashes  3  weeks  
aWer  starLng  Allopurinol  
B. 50/M  feels  light-­‐headed  with  rashes  aber  
taking  Co-­‐amoxiclav.  BP  is  60/40  
C. 18/M  with  hives  within  minutes  aWer  eaLng  
crabs  
D. 28/F  OR  nurse  with  rashes  on  her  hands  
usually  noted  aWer  her  duty  
Anaphylaxis  
•  Hallmark  of  AnaphylacLc  ReacLon:  Onset  of  
manifestaNons  within  seconds  to  minutes  
aber  introducNon  of  anNgen.  
•  Treatment:  0.3-­‐0.5  ml  of  Epinephrine  1:1000  
IM  or  SC;  2.5  ml  of  Epinephrine  1:10,000  IV  
88.  Which  of  the  following  diseases  is  the  most  
likely  diagnosis  for  these  spinal  tap  findings?  
High  leukocyte  count  (up  to  1000/L),  with  a  
predominance  of  lymphocytes  a  protein  content  
of  1–8  g/L  (100–800  mg/dL)  and  a  low  glucose  
concentraLon  
     A.  Herpes  simplex  encephaliLs  
     B.  Tuberculous  meningiLs  
     C.  Streptococcal  meningiLs  
     D.  Gonococcal  meningiLs  
88.  Which  of  the  following  diseases  is  the  most  
likely  diagnosis  for  these  spinal  tap  findings:  
high  leukocyte  count  (up  to  1000/L),  with  a  
predominance  of  lymphocytes  a  protein  content  
of  1–8  g/L  (100–800  mg/dL);  and  a  low  glucose  
concentraLon  

     A.  Herpes  simplex  encephaliLs  


     B.  Tuberculous  meningiNs  
     C.  Streptococcal  meningiLs  
     D.  Gonococcal  meningiLs  
89.    What  is  the  most  common  
cause  of  subarachnoid  hemorrhage  
A.      Bleeding  from  an  AV  malformaLon  
B.      Rupture  of  a  saccular  aneurysm  
C.      Head  trauma  
D.      Coagulopathy  
 
89.    What  is  the  most  common  
cause  of  subarachnoid  hemorrhage  
A.      Bleeding  from  an  AV  malformaLon  
B.      Rupture  of  a  saccular  aneurysm  
C.      Head  trauma  
D.      Coagulopathy  
 
90.    In  a  rabies  endemic  area  like  the  
Philippines,  the  diagnosis  of  Rabies  must  
be  considered  even  without  
 symptoms  of  hydrophobia  and  
aerophobia  with:  

A.      AsepLc  meningiLs  
B.    “Dumb  paralysis”  with  fever  
C.        Increased  intracranial  pressure  
D.      Hemiplegia  
 
90.    In  a  rabies  endemic  area  like  the  
Philippines,  the  diagnosis  of  Rabies  must  
be  considered  even  without  
 symptoms  of  hydrophobia  and  
aerophobia  with:  

A.      AsepLc  meningiLs  
B.    “Dumb  paralysis”  with  fever  
C.        Increased  intracranial  pressure  
D.      Hemiplegia  
 
91.  Kernig  sign  is  performed  and  
elicited  by:  
A. PaLent  in  supine  posiLon,  thigh  flexed,  knee  
flexed.    Pain  during  passive  extension  of  the  knee  
B. PaLent  in  supine  posiLon,  thigh  flexed,  knee  
extended.    Pain  during  passive  flexion  of  the  knee  
C. PaLent  in  supine  posiLon.    Passive  flexion  of  the  
neck  results  in  spontaneous  flexion  of  the  hips  
and  knees  
D. PaLent  in  supine  posiLon.  Passive  extension  of  
the  neck  results  in  spontaneous  extension  of  the  
hips  and  knees  
91.  Kernig  sign  is  performed  and  
elicited  by:  
A. PaNent  in  supine  posiNon,  thigh  flexed,  knee  
flexed.    Pain  during  passive  extension  of  the  knee  
B. PaLent  in  supine  posiLon,  thigh  flexed,  knee  
extended.    Pain  during  passive  flexion  of  the  knee  
C. PaLent  in  supine  posiLon.    Passive  flexion  of  the  
neck  results  in  spontaneous  flexion  of  the  hips  
and  knees  
D. PaLent  in  supine  posiLon.  Passive  extension  of  
the  neck  results  in  spontaneous  extension  of  the  
hips  and  knees  
92.  A  55  year  old  male,  previously  
diagnosed  with  a  glioblastoma  mulLforme  
came  in  for  decreased  sensorium.    Imaging  
revealed  perilesional  edema.    What  is  the  
appropriate  treatment?  
A. Furosemide  
B. Hypertonic  saline  
C. HydrocorLsone  
D. Dexamethasone  
 
92.  A  55  year  old  male,  previously  
diagnosed  with  a  glioblastoma  mulLforme  
came  in  for  decreased  sensorium.    Imaging  
revealed  perilesional  edema.    What  is  the  
appropriate  treatment?  
A. Furosemide  
B. Hypertonic  saline  
C. HydrocorLsone  
D. Dexamethasone  
 
93.  What  feature  would  favor  
seizures  over  syncope?  
A. No  precipitaLng  factor  
B. DuraLon  of  unconsciousness:  seconds  
C. Less  than  15  seconds  duraLon  of  tonic  
movements  
D. DuraLon  of  disorientaLon  aWer  the  
event:  less  than  5  minutes  
 
93.  What  feature  would  favor  
seizures  over  syncope?  
A. No  precipitaNng  factor  
B. DuraLon  of  unconsciousness:  seconds  
C. Less  than  15  seconds  duraLon  of  tonic  
movements  
D. DuraLon  of  disorientaLon  aWer  the  
event:  less  than  5  minutes  
 
Syncope  versus  Seizure  
SYNCOPE   SEIZURE  
•  Less  common  myoclonus   •  Typically  with  myoclonic  
•  Shorter  loss  of   jerks  
consciousness   •  Loss  of  consciousness  more  
than  5  minutes  
•  Less  muscle  aches  post-­‐
•  Post-­‐ictal  confusion  
asack  
•  Rarely  preceded  by  emoLon  
•  Can  have  urinary   or  pain  
inconLnence   •  Urinary  and  fecal  
•  Autonomic  manifestaLons   inconLnence  
•  Autonomic  manifestaLons  
94.  A  45  year  old  male  sought  consult  at  OPD  
because  of  mulLple  sharply  demarcated,  
erythematous  plaques  with  scaling  located  
predominantly  in  the  elbows  and  knees  as  
well  as  occasional  shoulder  and  knee  pains.  
What  is  your  diagnosis?  
A.  Lichen  planus  
B.  Pityriasis  rosea  
C.  Psoriasis  
D.  Tinea  versicolor  
94.  A  45  year  old  male  sought  consult  at  OPD  
because  of  mulLple  sharply  demarcated,  
erythematous  plaques  with  scaling  located  
predominantly  in  the  elbows  and  knees  as  
well  as  occasional  shoulder  and  knee  pains.  
What  is  your  diagnosis?  

A.  Lichen  planus  
B.  Pityriasis  rosea  
C.  Psoriasis  
D.  Tinea  versicolor  
95.  Which  primary  skin  lesions  is  
correctly  paired  with  its  descripLon?  
A.  Macule:  Flat,  colored  lesion,  <2cm  in  
diameter,  raised  above  the  surface  of  the  
surrounding  skin  
B.  Tumor:  >5  cm  firm  lesion  raised  above  the  
surface  of  the  surrounding  skin  
C.  Plaque:  >1cm,  flat-­‐topped,  raised  lesion  with  
edges  that  are  always  disLnctly  demarcated  
D.  Vesicle:  Small,  fluid-­‐filled  lesion,  >0.5cm  in  
diameter,  raised  above  the  plane  of  
surrounding  skin  
95.  Which  primary  skin  lesions  is  
correctly  paired  with  its  descripLon?  
A.  Macule:  Flat,  colored  lesion,  <2cm  in  
diameter,  raised  above  the  surface  of  the  
surrounding  skin  
B.  Tumor:  >5  cm  firm  lesion  raised  above  the  
surface  of  the  surrounding  skin  
C.  Plaque:  >1cm,  flat-­‐topped,  raised  lesion  with  
edges  that  are  always  disLnctly  demarcated  
D.  Vesicle:  Small,  fluid-­‐filled  lesion,  >0.5cm  in  
diameter,  raised  above  the  plane  of  
surrounding  skin  
Primary  Skin  Lesions  
96.    True  regarding  Type  2  Lepra  
reacLon  (erythema  nodosum  
leprosum  or  ENL)  
       A.    The  most  dramaLc  manifestaLon  is  
footdrop  
 B.    Edema  is  the  most  characterisLc  
microscopic  feature  
 C.    Fever  is  not  common  
 D.      Other  symptoms  include  neuriLs,  
lymphadeniLs,  uveiLs,  orchiLs,  and  
glomerulonephriLs  
96.    True  regarding  Type  2  Lepra  
reacLon  (erythema  nodosum  
leprosum  or  ENL)  
       A.    The  most  dramaLc  manifestaLon  is  
footdrop  
 B.    Edema  is  the  most  characterisLc  
microscopic  feature  
 C.    Fever  is  not  common  
 D.      Other  symptoms  include  neuriNs,  
lymphadeniNs,  uveiNs,  orchiNs,  and  
glomerulonephriNs  
Lepra  reacLons  
Type  1   Type  2  or  ENL  
•  Occurs  in  borderline  forms  of   •  Lepromatous  end  of  the  
leprosy   leprosy  spectrum    
•  Classic  signs  of  inflammaLon   •  Crops  of  painful  erythematous  
within  previously  involved   papules  that  resolve  
macules,  papules,  and  plaques     spontaneously  in  a  few  days  
•  Most  dramaLc  manifestaLon   •  Profound  fever,  neuriLs,  
is  footdrop   lymphadeniLs,  uveiLs,  orchiLs,  
•  Edema  is  the  most   and  glomerulonephriLs  
characterisLc  microscopic   •  Skin  biopsy  of  ENL  papules  
feature     reveals  vasculiLs  or  
panniculiLs  
•  Anemia,  leukocytosis,  and  
abnormal  liver  funcLon  tests    
97.  This  test  is  performed  on  
scaling  skin  lesions  when  a  fungal  
infecLon  is  suspected  
A. Tzanck  Smear  
B. Diascopy  
C. KOH  preparaLon  
D. Patch  tesLng  
 
97.  This  test  is  performed  on  
scaling  skin  lesions  when  a  fungal  
infecLon  is  suspected  
A. Tzanck  Smear  
B. Diascopy  
C. KOH  preparaNon  
D. Patch  tesLng  
 
98.    An  example  of  a  category  A  
bioterrorist  agent  

A.    E.  coli  0157:H7  


B.    Extremely-­‐drug  resistant  (XDR)  M.  
tuberculosis  
C.    SARS  Coronavirus  
D.    Variola  major  
 
98.    An  example  of  a  category  A  
bioterrorist  agent  

A.    E.  coli  0157:H7  


B.    Extremely-­‐drug  resistant  (XDR)  M.  
tuberculosis  
C.    SARS  Coronavirus  
D.    Variola  major  
 
Microbial  Bioterrorism  
A   Anthrax          Botulism        Plague        Smallpox        Tularemia  
Arenaviruses:  Lassa,  New  World  (Machupo,  Junin,  Guanarito,  
and  Sabia)  
Bunyaviridae:  Crimean-­‐Congo,  RiW  Valley  
Filoviridae:  Ebola,  Marburg  
B   Brucellosis      
Epsilon  toxin  of  Clostridium  perfringens    Glanders  Melioidosis  
Psisacosis  Q  fever      
Ricin  toxin  from  Ricinus  communis    Typhus  fever  
Staphylococcal  enterotoxin  B  Viral  encephaliLs  
Water  safety  threats  (e.g.,  Vibrio  cholerae,  Cryptosporidium  
parvum)    
Food  safety  threats  (e.g.,  Salmonella  spp.,  Escherichia  coli  
0157:H7,  Shigella)  
C   Emerging  infecLous  diseases  threats  such  as  Nipah,  hantavirus,  
SARS  coronavirus,  and  pandemic  influenza  
99.    Best  prevenLve  measure  
for  high  alLtude  pulmonary  
edema  
A.    Acetazolamide  
B.    Nifedipine  
C      LimitaLon  of  fluid  intake  prior  to  
ascent  
D.    Gradual  ascent  
99.    Best  prevenLve  measure  
for  high  alLtude  pulmonary  
edema  
A.    Acetazolamide  
B.    Nifedipine  
C      LimitaLon  of  fluid  intake  prior  to  
ascent  
D.    Gradual  ascent  
100.    Mrs  C  is  a  48  y/o  female  who  consulted  
you  a  few  years  back  for  dyspepsia.    She  
comes  to  your  clinic  now  with  easy  
faLgability,  weakness,  incoordinaLon  and  
memory  disturbances.    She  claims  she  cannot  
tolerate  eaLng  without  taking  antacids.    
Physical  and  neurologic  exams  are  
unremarkable.    Among  the  labs  you  ordered,  
only  her  CBC  is  abnormal  showing  red  blood  
cells  with  high  MCV  and  high  MCH.      
100.  Which  of  the  following  
nutrients  is  she  most  likely  
deficient  with?  
A.    Iron  
B.    Folate  
C.    Cyanocobalamin  
D.    Vitamin  D  
 
100.  Which  of  the  following  
nutrients  is  she  most  likely  
deficient  with?  
A.    Iron  
B.    Folate  
C.    Cyanocobalamin  
D.    Vitamin  D  
 
101.  Cause  of  normal  anion  gap  
metabolic  acidosis:  

A.  Diarrhea  
B.  Alcoholic  ketoacidosis  
C.  Uremia  
D.  Salicylate  ingesLon  
101.  Cause  of  normal  anion  gap  
metabolic  acidosis:  

A.  Diarrhea  
B.  Alcoholic  ketoacidosis  
C.  Uremia  
D.  Salicylate  ingesLon  
102.    Which  of  the  following  is  a  risk  
factor  for  stroke  in  a  paLent  with  atrial  
fibrillaLon?    

A.  Age  more  than  65  years  old  


B.  Mitral  regurgitaLon    
C.  Diabetes  mellitus  
D.  Marked  right  atrial  enlargement  
 
102.    Which  of  the  following  is  a  risk  
factor  for  stroke  in  a  paLent  with  atrial  
fibrillaLon?    

A.  Age  more  than  65  years  old  


B.  Mitral  regurgitaLon    
C.  Diabetes  mellitus  
D.  Marked  right  atrial  enlargement  
 
Risk  Factors  for  Stroke  in    
Atrial  FibrillaNon  
•  History  of  stroke  or  transient  ischemic  asack    
•  Mitral  stenosis    
•  Hypertension  
•  Diabetes  mellitus  
•  Age  >75  years    
•  CongesLve  heart  failure    
•  LeW  ventricular  dysfuncLon    
•  Marked  leW  atrial  enlargement  (>5.0  cm)    
•  Spontaneous  echo  contrast  
103.    True  of  manifestaLons  of  
myxomas    
A.  Most  appear  singly  
B.  Most  are  pedunculated  on  a  
fibrovascular  stalk  
C.    Most  are  located  in  the  leW  atrium  
D.  They  have  a  characterisLc  “tumor  plop”  
appreciated  during  diastole  
103.    True  of  manifestaLons  of  
myxomas    
A.  Most  appear  singly  
B.  Most  are  pedunculated  on  a  
fibrovascular  stalk  
C.    Most  are  located  in  the  leW  atrium  
D.  They  have  a  characterisLc  “tumor  plop”  
appreciated  during  diastole  
104.  Which  of  the  following  
medicaLons  should  be  not  be  started  
in  paLents  with  hyperkalemia?  

A.  Carvedilol  
B.  Dobutamine  
C.  Captopril  
D.  Furosemide  
 
104.  Which  of  the  following  
medicaLons  should  be  not  be  started  
in  paLents  with  hyperkalemia?  

A.  Carvedilol  
B.  Dobutamine  
C.  Captopril  
D.  Furosemide  
 
105.  For  angina,  nitrates  may  be  
administered.    Nitroglycerin  is  most  
commonly  administered  sublingually  
in  what  dose    
A.  5  mg  
B.  10  mg  
C.  0.4  mg  
D.  4  mg  
105.  For  angina,  nitrates  may  be  
administered.    Nitroglycerin  is  most  
commonly  administered  sublingually  
in  what  dose    
A.  5  mg  
B.  10  mg  
C.  0.4  mg  
D.  4  mg  
106.  A  19  year  old  female  comes  in  for  
dyspnea.    As  a  child,  she  had  cyanosis,  
selecLvely  involving  the  toes  only  (but  
not  the  fingers).    She  was  said  to  have  a  
“defect  in  her  heart”,  but  was  lost  to  
follow  up.    On  auscultaLon,  you  should  
expect:  
A.  A  fixed  splizng  of  S2    
B.  A  holosystolic  murmur  
C.  A  conLnuous  machinery  murmur  
D.  Early  systolic  murmur  
106.  A  19  year  old  female  comes  in  for  
dyspnea.    As  a  child,  she  had  cyanosis,  
selecLvely  involving  the  toes  only  (but  
not  the  fingers).    She  was  said  to  have  a  
“defect  in  her  heart”,  but  was  lost  to  
follow  up.    On  auscultaLon,  you  should  
expect:  
A.  A  fixed  splizng  of  S2    
B.  A  holosystolic  murmur  
C.  A  conNnuous  machinery  murmur  
D.  Early  systolic  murmur  
107.  The  most  common  congenital  
heart  valve  defect  is:  

A.  Pulmonic  stenosis  


B.  Mitral  stenosis  
C.  Bicuspid  aorLc  valve  
D.  Mitral  regurgitaLon  
107.  The  most  common  congenital  
heart  valve  defect  is:  

A.  Pulmonic  stenosis  


B.  Mitral  stenosis  
C.  Bicuspid  aorNc  valve  
D.  Mitral  regurgitaLon  
108.  A  40  year  old  female  came  in  for  dyspnea.    
Blood  pressure  on  admission  was  120  /20  
mmHg.    AuscultaLon  revealed  a  high-­‐pitched,  
blowing,  decrescendo  diastolic  murmur  at  the  
right  sternal  border.    You  also  noted  capillary  
pulsaLons  (alternate  flushing  and  paling  of  the  
skin)  at  the  root  of  the  nail  while  pressure  is  
applied  to  the  Lp  of  the  nail.    This  is  called:  
 
A.  Corrigan’s  pulse  
B.  Quincke’s  pulse  
C.  Traube’s  sign  
D.  Duroziez’s  sign  
108.  A  40  year  old  female  came  in  for  dyspnea.    
Blood  pressure  on  admission  was  120  /20  
mmHg.    AuscultaLon  revealed  a  high-­‐pitched,  
blowing,  decrescendo  diastolic  murmur  at  the  
right  sternal  border.    You  also  noted  capillary  
pulsaLons  (alternate  flushing  and  paling  of  the  
skin)  at  the  root  of  the  nail  while  pressure  is  
applied  to  the  Lp  of  the  nail.    This  is  called:  
 
A.  Corrigan’s  pulse  
B.  Quincke’s  pulse  
C.  Traube’s  sign  
D.  Duroziez’s  sign  
AorLc  RegurgitaLon  
•  De  Musset  sign:  head  may  bob  with  each  heartbeat  
•  Corrigan  pulse:  water  hammer  pulses  with  abrupt  
distenLon  and  quick  collapse  
•  Traube  sign:  pistol  shot  sounds  –  booming  systolic  and  
diastolic  sounds  heard  over  femoral  artery  
•  Muller  sign:  systolic  pulsaLon  of  uvula  
•  Duroziez  sign:  systolic  murmur  heard  over  femoral  
artery  when  compressed  proximally  and  a  diastolic  
murmur  when  compressed  distally  
•  Hill  sign:  exaggerated  difference  in  SBP  between  upper  
and  lower  extremiLes    
109.  A  35  year  old  male  comes  in  for  dyspnea  
and  orthopnea.    On  examinaLon,  she  had  a  
heart  rate  of  110  with  an  irregularly  irregular  
rhythm.    An  opening  snap,  followed  by  a  
diastolic  rumble  was  audible  on  auscultaLon.    
You  requested  a  chest  x-­‐ray  and  expect  to  find  
A.  Widening  of  the  carinal  angle  
B.  LeW  ventricular  enlargement  
C.  Tubular  heart  with  a  widened  
mediasLnum  
D.  Paucity  of  pulmonary  vasculature  
109.  A  35  year  old  male  comes  in  for  dyspnea  
and  orthopnea.    On  examinaLon,  she  had  a  
heart  rate  of  110  with  an  irregularly  irregular  
rhythm.    An  opening  snap,  followed  by  a  
diastolic  rumble  was  audible  on  auscultaLon.    
You  requested  a  chest  x-­‐ray  and  expect  to  find  
A.  Widening  of  the  carinal  angle  
B.  LeW  ventricular  enlargement  
C.  Tubular  heart  with  a  widened  
mediasLnum  
D.  Paucity  of  pulmonary  vasculature  
110.  Most  common  parasiLc  cause  of  
cardiomyopathy  is  Chagas’  disease.    
What  is  the  eLology?  
 
A.  Trypanasoma  cruzi  transmised  by  the  
reduviid  bug  
B.  Trypanosoma  brucei  transmised  by  the  
Tsetse  fly  
C.  Toxoplasmosis  transmised  by  cysts  
D.  Trichinella  spiralis  caused  by  ingesLon  of  
larva  of  undercooked  meat  
110.  Most  common  parasiLc  cause  of  
cardiomyopathy  is  Chagas’  disease.    
What  is  the  eLology?  
 
A.  Trypanasoma  cruzi  transmired  by  the  
reduviid  bug  
B.  Trypanosoma  brucei  transmised  by  the  
Tsetse  fly  
C.  Toxoplasmosis  transmised  by  cysts  
D.  Trichinella  spiralis  caused  by  ingesLon  of  
larva  of  undercooked  meat  
111.  A  35  year  old  male  came  in  for  
dyspnea.    He  had  a  blood  pressure  of  
60/40mmHg  with  muffled  heart  
sounds  and  distended  neck  veins.    
Your  acute  management  is:  
 
A.  Immediately  start  inotropic  support  
B.  Place  in  Trendelenberg  posiLon  and  
observe  
C.  Give  furosemide  40mg  IV  stat  bolus  
D.  Do  2D  echo-­‐guided  pericardiocentesis  
111.  A  35  year  old  male  came  in  for  
dyspnea.    He  had  a  blood  pressure  of  
60/40mmHg  with  muffled  heart  
sounds  and  distended  neck  veins.    
Your  acute  management  is:  
 
A.  Immediately  start  inotropic  support  
B.  Place  in  Trendelenberg  posiLon  and  
observe  
C.  Give  furosemide  40mg  IV  stat  bolus  
D.  Do  2D  echo-­‐guided  pericardiocentesis  
112.  Most  out-­‐of  hospital  deaths  
from  STEMI  is  due  to  
 
A.  Heart  failure  
B.  Ventricular  fibrillaLon  
C.  Premature  ventricular  contracLons  
D.  Pulmonary  edema  
112.  Most  out-­‐of  hospital  deaths  
from  STEMI  is  due  to  
 
A.  Heart  failure  
B.  Ventricular  fibrillaNon  
C.  Premature  ventricular  contracLons  
D.  Pulmonary  edema  
113.    A  22  year  old  male  will  be  working  abroad  as  
a  construcLon  worker  in  Dubai.  This  paLent  came  
in  to  your  clinic  for  clearance.  He  is  asymptomaLc.  
On  PE,  you  noted  grade  II/VI  midsystolic  murmur  at  
the  apex.  Chest  x-­‐ray  and  ECG  were  normal.  What  
would  be  your  next  step?    
A.  Reassure  and  clear  him     for  work,  no  further  tests  
B.  Delay  giving  of  clearance  and  schedule  for  2D  
echocardiography  
C.  Refer  to  cardiologist  for  clearance  
D.  Admit  for  further  work-­‐up:  Holter  monitoring,  
cardiac  enzymes  and  possible  coronary  
angiography  
113.    A  22  year  old  male  will  be  working  abroad  as  
a  construcLon  worker  in  Dubai.  This  paLent  came  
in  to  your  clinic  for  clearance.  He  is  asymptomaLc.  
On  PE,  you  noted  grade  II/VI  midsystolic  murmur  at  
the  apex.  Chest  x-­‐ray  and  ECG  were  normal.  What  
would  be  your  next  step?    
A.  Reassure  and  clear  him     for  work,  no  further  
tests  
B.  Delay  giving  of  clearance  and  schedule  for  2D  
echocardiography  
C.  Refer  to  cardiologist  for  clearance  
D.  Admit  for  further  work-­‐up:  Holter  monitoring,  
cardiac  enzymes  and  possible  coronary  
angiography  
114.  What  is  a  relaLve  contraindicaLon  
of  thrombolysis  in  ST-­‐  segment  elevaLon  
myocardial  infarcLon?  
 
A.  History  of  cerebrovascular  
hemorrhage  
B.  Suspicion  of  aorLc  dissecLon  
C.  AcLve  internal  bleeding  
D.  Pregnancy  
114.  What  is  a  relaLve  contraindicaLon  
of  thrombolysis  in  ST-­‐  segment  elevaLon  
myocardial  infarcLon?  
 
A.  History  of  cerebrovascular  
hemorrhage  
B.  Suspicion  of  aorLc  dissecLon  
C.  AcLve  internal  bleeding  
D.  Pregnancy  
115.  Presents  as  refractory  
hypertension,  mostly  asymptomaLc  
but  infrequently  may  have  
paresthesias,  polyuria,  or  muscle  
weakness  secondary  to  hypokalemic  
alkalosis    
A.  Cushing’s  syndrome    
B.  Pheochromocytoma  
C.  Primary  aldosteronism  
D.  AorLc  coarctaLon  
115.  Presents  as  refractory  
hypertension,  mostly  asymptomaLc  
but  infrequently  may  have  
paresthesias,  polyuria,  or  muscle  
weakness  secondary  to  hypokalemic  
alkalosis    
A.  Cushing’s  syndrome    
B.  Pheochromocytoma  
C.  Primary  aldosteronism  
D.  AorLc  coarctaLon  
116.  Abdominal  aorLc  aneurysm  
repair  is  indicated  for  asymptomaLc  
paLents  if  the  diameter  is:  

A. >  4.5cm  
B. >  5.5  cm  
C. >  6.0  cm  
D. >  7.0  cm  
116.  Abdominal  aorLc  aneurysm  
repair  is  indicated  for  asymptomaLc  
paLents  if  the  diameter  is:  

A. >  4.5cm  
B.  >  5.5  cm  
C. >  6.0  cm  
D. >  7.0  cm  
117.  Which  of  the  following  physical  
examinaLon  findings,  if  present  in  
COPD  suggest  advanced  disease?  

A.  Barrel  chest  
B.  Temporal  wasLng    
C.  Clubbing  of  the  digits  
D.  Cyanosis  
117.  Which  of  the  following  physical  
examinaLon  findings,  if  present  in  
COPD  suggest  advanced  disease?  

A.  Barrel  chest  
B.  Temporal  wasNng    
C.  Clubbing  of  the  digits  
D.  Cyanosis  
118.  The  standard  monotherapy  
for  lung  abscess  

A.  CeWriaxone  
B.  Metronidazole  
C.  Clindamycin  
D.  Ampicillin-­‐Sulbactam  
118.  The  standard  monotherapy  
for  lung  abscess  

A.  CeWriaxone  
B.  Metronidazole  
C.  Clindamycin  
D.  Ampicillin-­‐Sulbactam  
119.    Which  of  the  following  points  to  
an  exacerbaLon  in  a  paLent  with  
established  COPD  

A.  ExerLonal  dyspnea  
B.  Increased  dyspnea  
C.  Presence  of  airflow  obstrucLon  
D.  ResLng  hypoxemia  
 
119.    Which  of  the  following  points  to  
an  exacerbaLon  in  a  paLent  with  
established  COPD  

A.  ExerLonal  dyspnea  
B.  Increased  dyspnea  
C.  Presence  of  airflow  obstrucLon  
D.  ResLng  hypoxemia  
 
120.    The  following  are  true  regarding  
non-­‐imaging  tests  in  diagnosing  
pulmonary  embolism  EXCEPT:  

A.  The  D-­‐dimer  is  a  useful  "rule  in"  test  


B.  The  D-­‐dimer  has  a  limited  role  in  hospitalized  
paLents  
C.  Arterial  blood  gas  lacks  diagnosLc  uLlity  for  
pulmonary  embolism  
D.  Elevated  cardiac  biomarkers  predict  an  
increased  mortality  from  pulmonary  embolism  
120.    The  following  are  true  regarding  
non-­‐imaging  tests  in  diagnosing  
pulmonary  embolism  EXCEPT:  

A.  The  D-­‐dimer  is  a  useful  "rule  in"  test  


B.  The  D-­‐dimer  has  a  limited  role  in  hospitalized  
paLents  
C.  Arterial  blood  gas  lacks  diagnosLc  uLlity  for  
pulmonary  embolism  
D.  Elevated  cardiac  biomarkers  predict  an  
increased  mortality  from  pulmonary  embolism  
121.  When  aWer  iniLaLon  of  adequate  
therapy  in  pneumonia  do  you  expect  
fever  to  resolve?  

A.  24  hours  
B.  36  hours  
C.  48  hours  
D.  72  hours  
 
121.  When  aWer  iniLaLon  of  adequate  
therapy  in  pneumonia  do  you  expect  
fever  to  resolve?  

A.  24  hours  
B.  36  hours  
C.  48  hours  
D.  72  hours  
 
122.    Among  COPD  paLents,  
supplemental  oxygen  should  be  
provided  to  maintain  arterial  oxygen  
saturaLon  at  what  level?  

A.  80%  
B.  85%  
C.  90%  
D.  95%  
 
122.    Among  COPD  paLents,  
supplemental  oxygen  should  be  
provided  to  maintain  arterial  oxygen  
saturaLon  at  what  level?  

A.  80%  
B.  85%  
C.  90%  
D.  95%  
 
IndicaNons  for  Supplemental  O2  in  
COPD  

•  ResLng  O2  sat  <  88%  


•  ResLng  O2  sat  <  90%  in  those  with  
pulmonary  hypertension  and  right  heart  
failure  
123.  OccupaLonal  lung  disease  
characterized  by  the  characterisLc  
HRCT  pasern  known  as  “crazy  paving”  

A.  Chronic  beryllium  disease  


B.  Coal  worker’s  pneumoconiosis  
C.  Asbestosis  
D.  Silicosis    
123.  OccupaLonal  lung  disease  
characterized  by  the  characterisLc  
HRCT  pasern  known  as  “crazy  paving”  

A.  Chronic  beryllium  disease  


B.  Coal  worker’s  pneumoconiosis  
C.  Asbestosis  
D.  Silicosis    
124.  What  is  the  most  common  cause  
of  respiratory  hypoxia,  which  is  usually  
correctable  by  inspiring  100%  oxygen?  

A.  HypovenLlaLon  
B.  Intrapulmonary  right  to  leW  shunLng  
C.  Pulmonary  atelectasis  
D.  VenLlaLon  perfusion  mismatch  
124.  What  is  the  most  common  cause  
of  respiratory  hypoxia,  which  is  usually  
correctable  by  inspiring  100%  oxygen?  

A.  HypovenLlaLon  
B.  Intrapulmonary  right  to  leW  shunLng  
C.  Pulmonary  atelectasis  
D.  VenNlaNon  perfusion  mismatch  
125.  Which  among  the  following  is  
an  example  of  obstrucLve  lung  
disease?  
A.  Bronchiectasis  
B.  Asbestosis  
C.  Pulmonary  fibrosis  
D.  Guillain-­‐Barre  syndrome  
125.  Which  among  the  following  is  
an  example  of  obstrucLve  lung  
disease?  
A.  Bronchiectasis  
B.  Asbestosis  
C.  Pulmonary  fibrosis  
D.  Guillain-­‐Barre  syndrome  
126.  An  asthmaLc  paLent  was  given  
ipratropium  bromide  for  rescue  
medicaLon.    What  is  its  most  common  
side  effect?  

A.  Tachycardia  or  palpitaLons  


B.  Tremors  
C.  Dry  mouth  
D.  Urinary  retenLon  
126.  An  asthmaLc  paLent  was  given  
ipratropium  bromide  for  rescue  
medicaLon.    What  is  its  most  common  
side  effect?  

A.  Tachycardia  or  palpitaLons  


B.  Tremors  
C.  Dry  mouth  
D.  Urinary  retenLon  
127.  What  is  the  most  common  
cancer  associated  with  asbestos  
exposure?  

A.  Lung  cancer  


B.  Mesothelioma  
C.  Tumors  of  the  pericardium  
D.  Laryngeal  carcinoma  
127.  What  is  the  most  common  
cancer  associated  with  asbestos  
exposure?  

A.  Lung  cancer  


B.  Mesothelioma  
C.  Tumors  of  the  pericardium  
D.  Laryngeal  carcinoma  
128.  A  60  year  old  male  presented  
with  cough  and  dyspnea  of  three  days  
duraLon.    You  requested  for  a  sputum  
gram  stain  and  culture.    An  adequate  
sputum  sample  has:  

A.  Presence  of  organisms  


B.  Presence  of  >  25  neutrophils  
C.  Presence  of  >  25  squamous  cells  
D.  Presence  of  >  25  white  blood  cells  
128.  A  60  year  old  male  presented  
with  cough  and  dyspnea  of  three  days  
duraLon.    You  requested  for  a  sputum  
gram  stain  and  culture.    An  adequate  
sputum  sample  has:  

A.  Presence  of  organisms  


B.  Presence  of  >  25  neutrophils  
C.  Presence  of  >  25  squamous  cells  
D.  Presence  of  >  25  white  blood  cells  
129.  Type  of  emphysema  usually  
observed  in  paLents  with  alpha-­‐1-­‐
anLtrypsin  deficiency,  which  has  a  
predilecLon  for  the  lower  lobes:  
A.  Centriacinar  emphysema  
B.  Peripheral  emphysema  
C.  Lobar  emphysema  
D.  Panacinar  emphysema  
129.  Type  of  emphysema  usually  
observed  in  paLents  with  alpha-­‐1-­‐
anLtrypsin  deficiency,  which  has  a  
predilecLon  for  the  lower  lobes:  
A.  Centriacinar  emphysema  
B.  Peripheral  emphysema  
C.  Lobar  emphysema  
D.  Panacinar  emphysema  
130.  During  the  natural  course  of  
ARDS,  which  phase  do  we  expect  
clinical  recovery  wherein  paLents  are  
usually  liberated  from  mechanical  
venLlaLon?  

A.  ProliferaLve  phase  


B.  FibroLc  phase  
C.  Recovery  phase  
D.  ExudaLve  phase  
130.  During  the  natural  course  of  
ARDS,  which  phase  do  we  expect  
clinical  recovery  wherein  paLents  are  
usually  liberated  from  mechanical  
venLlaLon?  

A.  ProliferaNve  phase  


B.  FibroLc  phase  
C.  Recovery  phase  
D.  ExudaLve  phase  
131.  Chest  radiograph  finding  
specifically  indicaLng  past  exposure  to  
asbestos  and  not  pulmonary  
impairment:  

A.  Pleural  plaque  


B.  Pleural  effusion  
C.  Pleural  fibrosis  
D.  Pleural  mesothelioma  
131.  Chest  radiograph  finding  
specifically  indicaLng  past  exposure  to  
asbestos  and  not  pulmonary  
impairment:  

A.  Pleural  plaque  


B.  Pleural  effusion  
C.  Pleural  fibrosis  
D.  Pleural  mesothelioma  
132.  A  30  year  old  male  comes  in  for  
dyspnea.    ExaminaLon  on  the  right  
basal  lung  field  revealed:  dullness  on  
percussion,  increased  fremitus,  and  
crackles.    Your  diagnosis  is  probably:  

A.  Asthma  
B.  Pneumothorax  
C.  Pleural  effusion  
D.  ConsolidaLon  /  pneumonia  
132.  A  30  year  old  male  comes  in  for  
dyspnea.    ExaminaLon  on  the  right  
basal  lung  field  revealed:  dullness  on  
percussion,  increased  fremitus,  and  
crackles.    Your  diagnosis  is  probably:  

A.  Asthma  
B.  Pneumothorax  
C.  Pleural  effusion  
D.  ConsolidaNon  /  pneumonia  
133.  The  following  cause  of  ARDS  is  
due  to  a  direct  lung  injury:  

A.  Sepsis  
B.  Chest  trauma  
C.  Near  drowning  
D.  MulLple  blood  transfusions  
133.  The  following  cause  of  ARDS  is  
due  to  a  direct  lung  injury:  

A.  Sepsis  
B.  Chest  trauma  
C.  Near  drowning  
D.  MulLple  blood  transfusions  
ARDS  
134.  What  is  the  prevailing  
mechanism  for  the  development  of  
emphysema?  

A.  Dutch  hypothesis  


B.  BriLsh  hypothesis  
C.  Hygiene  hypothesis  
D.  Elastase:  AnL-­‐elastase  hypothesis  
134.  What  is  the  prevailing  
mechanism  for  the  development  of  
emphysema?  

A.  Dutch  hypothesis  


B.  BriLsh  hypothesis  
C.  Hygiene  hypothesis  
D.  Elastase:  AnN-­‐elastase  hypothesis  
135.  The  CURB-­‐65  criteria  is  used  in  
pneumonia  as  a  severity  of  illness  
score.    Which  is  INCORRECTLY  paired?  

A. C:  CreaLnine  >  250  


B. U:  Urea  >  7  mmol/L  
C. R:  Respiratory  rate  >  30/min  
D. B:  Blood  pressure  <  90/60  
135.  The  CURB-­‐65  criteria  is  used  in  
pneumonia  as  a  severity  of  illness  
score.    Which  is  INCORRECTLY  paired?  

A. C:  CreaNnine  >  250  


B. U:  Urea  >  7  mmol/L  
C. R:  Respiratory  rate  >  30/min  
D. B:  Blood  pressure  <  90/60  
CURB  
–  The  CURB-­‐65  criteria  include  five  variables:    
•  (C)  Confusion    
•  (U)  Urea  >7  mmol/L    
•  (R)  Respiratory  rate  30/min    
•  (B)    BP,  systolic  90  mmHg  or  diastolic  60  mmHg    
•  (65)  
 Age  65  years  
136.  What  is  the  most  common  
cause  of  hemoptysis  worldwide?  

A.  Pneumonia  
B.  Tuberculosis  
C.  Bronchogenic  carcinoma  
D.  Upper  respiratory  tract  infecLon  
136.  What  is  the  most  common  
cause  of  hemoptysis  worldwide?  

A.  Pneumonia  
B.  Tuberculosis  
C.  Bronchogenic  carcinoma  
D.  Upper  respiratory  tract  infecLon  
137.  A  60  year  old  male  was  previously  
diagnosed  with  tuberculosis  but  was  never  
compliant  to  medicaLons.    He  presented  
with  a  three  month  history  of  producLve  
cough  and  dyspnea.    Chest  x-­‐ray  revealed  
presence  of  tram  tracks.    What  is  your  
diagnosis?  
A.  Asbestosis    
B.  Emphysema  
C.  Bronchiectasis  
D.  Lung  abscess  
137.  A  60  year  old  male  was  previously  
diagnosed  with  tuberculosis  but  was  never  
compliant  to  medicaLons.    He  presented  
with  a  three  month  history  of  producLve  
cough  and  dyspnea.    Chest  x-­‐ray  revealed  
presence  of  tram  tracks.    What  is  your  
diagnosis?  
A.  Asbestosis    
B.  Emphysema  
C.  Bronchiectasis  
D.  Lung  abscess  
138.  A  45  year  old  female  with  an  ovarian  
malignancy  comes  in  for  dyspnea  and  hemoptysis.    
On  examinaLon,  she  was  tachycardic  and  
tachypneic,  but  with  stable  blood  pressure.    Her  
enLre  leW  lower  extremity  is  swollen  and  tender  
with  erythema.    To  confirm  your  diagnosis  you  
would  request  for:  
 
A.  D-­‐dimer  
B.  Pelvic  CT  scan  
C.  Chest  CT  scan  
D.  Venous  duplex  scan,  lower  extremity  
138.  A  45  year  old  female  with  an  ovarian  
malignancy  comes  in  for  dyspnea  and  hemoptysis.    
On  examinaLon,  she  was  tachycardic  and  
tachypneic,  but  with  stable  blood  pressure.    Her  
enLre  leW  lower  extremity  is  swollen  and  tender  
with  erythema.    To  confirm  your  diagnosis  you  
would  request  for:  
 
A.  D-­‐dimer  
B.  Pelvic  CT  scan  
C.  Chest  CT  scan  
D.  Venous  duplex  scan,  lower  extremity  
139.  Most  common  mode  of  
transmission  of  HIV  in  developing  
countries  such  as  the  Philippines  

A.    Perinatal  transfer  
B.    Homosexual  transmission  
C.    Heterosexual  transmission  
D.    Contaminated  IV  drug  paraphernalia  
 
139.  Most  common  mode  of  
transmission  of  HIV  in  developing  
countries  such  as  the  Philippines  

A.    Perinatal  transfer  
B.    Homosexual  transmission  
C.    Heterosexual  transmission  
D.    Contaminated  IV  drug  paraphernalia  
 
140.  The  heart  valve  most  
commonly  affected  with  infecLve  
endocardiLs  among  IV  drug  users    
     A.  Mitral  valve  
     B.  Tricuspid  valve  
     C.  Pulmonic  valve  
     D.  AorLc  valve  
 
140.  The  heart  valve  most  
commonly  affected  with  infecLve  
endocardiLs  among  IV  drug  users    
     A.  Mitral  valve  
     B.  Tricuspid  valve  
     C.  Pulmonic  valve  
     D.  AorLc  valve  
 
141.  Most  common  clinical  
manifestaLon  of  InfecLve  
EndocardiLs:  

     A.    Fever  
     B.    Chills  
     C.    Heart  murmur  
     D.  Janeway  lesions  
 
141.  Most  common  clinical  
manifestaLon  of  InfecLve  
EndocardiLs:  

     A.    Fever  
     B.    Chills  
     C.    Heart  murmur  
     D.  Janeway  lesions  
 
142.    True  about  the  diagnosis  of  
InfecLve  EndocardiLs  (IE):  
     A.    Diagnosis  is  based  on  the  Jones  Criteria  
     B.    Organisms  can  oWen  be  cultured  from  blood  
despite  prior  anLbioLc  therapy  
     C.    Diagnosis  of  IE  is  rejected  if  fever  and  
symptoms  resolve  in  less  than  4  days  of  
therapy  
     D.    Typical  organisms  recovered  in  blood  culture  
are  E.coli  and  S.viridans  
   
142.    True  about  the  diagnosis  of  
InfecLve  EndocardiLs  (IE):  
     A.    Diagnosis  is  based  on  the  Jones  Criteria  
     B.    Organisms  can  oWen  be  cultured  from  blood  
despite  prior  anLbioLc  therapy  
     C.    Diagnosis  of  IE  is  rejected  if  fever  and  
symptoms  resolve  in  less  than  4  days  of  
therapy  
     D.    Typical  organisms  recovered  in  blood  culture  
are  E.coli  and  S.viridans  
   
143.    Which  sexually  transmised  
disease  is  associated  with  
gummas?  
 
     A.  Syphilis  
     B.  Human  papilloma  virus  
     C.  HIV  
     D.  Donovanosis  
 
143.    Which  sexually  transmised  
disease  is  associated  with  
gummas?  
 
     A.  Syphilis  
     B.  Human  papilloma  virus  
     C.  HIV  
     D.  Donovanosis  
 
144.  Pasteurella  multocida  is  known  to  
commonly  infect  dog  and  cat  bites.  
What  is  the  anLbioLc  of  choice  for  this  
organism?  

     A.  Cloxacillin  
     B.  Penicillin  G  
     C.  Co-­‐amoxyclav  
     D.  Clindamycin  
 
144.  Pasteurella  multocida  is  known  to  
commonly  infect  dog  and  cat  bites.  
What  is  the  anLbioLc  of  choice  for  this  
organism?  

     A.  Cloxacillin  
     B.  Penicillin  G  
     C.  Co-­‐amoxyclav  
     D.  Clindamycin  
 
145.    Remains  the  standard  and  
recommended  duraLon  of  IV  
anLobioLc  therapy  for  osteomyeliLs  

     A.  2-­‐4  weeks  


     B.  4-­‐6  weeks  
     C.  8-­‐12  weeks  
     D.  3-­‐6  months  
   
145.    Remains  the  standard  and  
recommended  duraLon  of  IV  
anLobioLc  therapy  for  osteomyeliLs  

     A.  2-­‐4  weeks  


     B.  4-­‐6  weeks  
     C.  8-­‐12  weeks  
     D.  3-­‐6  months  
   
146.  AnLbioLc/s  that  may  be  used  
for  only  3  days  in  acute  
uncomplicated  cysLLs  in  women    

 A.  TMP-­‐SMX  
 B.  Nitrofurantoin  
 C.  Quinolones  
 D.  All  of  the  above  
 
146.  AnLbioLc/s  that  may  be  used  
for  only  3  days  in  acute  
uncomplicated  cysLLs  in  women    

 A.  TMP-­‐SMX  
 B.  Nitrofurantoin  
 C.  Quinolones  
 D.  All  of  the  above  
 
147.  DramaLc  but  mild  reacLon  
consisLng  of  fever,  chills,  myalgias,  
tachycardia  with  mild  hypotension  noted  
aWer  iniLaLon  of  anLbioLc  therapy  for  
leptospirosis  

A.  Lucio  reacLon  


B.  Weil’s  Syndrome  
C.  Nicoladoni  Branham  Sign  
D.  Jarisch-­‐Herxheimer  ReacLon  
147.  DramaLc  but  mild  reacLon  
consisLng  of  fever,  chills,  myalgias,  
tachycardia  with  mild  hypotension  noted  
aWer  iniLaLon  of  anLbioLc  therapy  for  
leptospirosis  

A.  Lucio  reacLon  


B.  Weil’s  Syndrome  
C.  Nicoladoni  Branham  Sign  
D.  Jarisch-­‐Herxheimer  ReacNon  
148.    Most  important  risk  factor  
associated  with  the  development  
of  perinephric  abscesses  
A.  Diabetes  mellitus  
B.  Prior  urologic  surgery  
C.  Chronic  kidney  disease  
D.  Nephrolithiasis  with  obstrucLon  
148.    Most  important  risk  factor  
associated  with  the  development  
of  perinephric  abscesses  
A.  Diabetes  mellitus  
B.  Prior  urologic  surgery  
C.  Chronic  kidney  disease  
D.  Nephrolithiasis  with  obstrucNon  
149.    Most  commonly  implicated  
organism/s  in  Primary  Bacterial  
PeritoniLs:  
     A.    Escherichia  coli  
     B.    Pseudomonas  aeruginosa  
     C.    Anaerobic  bacteria  
     D.    Mixed  infecLon  
 
149.    Most  commonly  implicated  
organism/s  in  Primary  Bacterial  
PeritoniLs:  
     A.    Escherichia  coli  
     B.    Pseudomonas  aeruginosa  
     C.    Anaerobic  bacteria  
     D.    Mixed  infecLon  
 
150.    Single  most  reliable  laboratory  
parameter  in  liver  abscess  

     A.  Increased  AST  and  ALT  


     B.  Increased  alkaline  phosphatase  
     C.  NormocyLc,  normochromic  anemia  
     D.  Increased  direct  bilibubin  
 
150.    Single  most  reliable  laboratory  
parameter  in  liver  abscess  

     A.  Increased  AST  and  ALT  


     B.  Increased  alkaline  phosphatase  
     C.  NormocyLc,  normochromic  anemia  
     D.  Increased  direct  bilibubin  
 
151.    Most  common  
extrapulmonary  manifestaLon  of  
tuberculosis  

     A.  TB  meningiLs  


     B.  TB  adeniLs  
     C.  Pos's  disease  
     D.  Genitourinary  TB  
 
151.    Most  common  
extrapulmonary  manifestaLon  of  
tuberculosis  

     A.  TB  meningiLs  


     B.  TB  adeniNs  
     C.  Pos's  disease  
     D.  Genitourinary  TB  
 
152.    In  which  tuberculous  infecLon  
are  systemic  steroids  like  
dexamethasone  proven  to  posiLvely  
influence  outcomes?  
     A.  TB  meningiLs  
     B.  Pos's  disease  
     C.  Tuberculous  pericardial  effusion  
     D.  Miliary  TB  
   
 
152.    In  which  tuberculous  infecLon  
are  systemic  steroids  like  
dexamethasone  proven  to  posiLvely  
influence  outcomes?  
     A.  TB  meningiNs  
     B.  Pos's  disease  
     C.  Tuberculous  pericardial  effusion  
     D.  Miliary  TB  
   
 
153.    The  single  most  important  
agent  of  Traveler's  diarrhea  

 A.  Enterotoxigenic  E.  coli  


 B.  Salmonella  typhi  
 C.  Campylobacter  jejuni  
 D.  Rotavirus  
 
153.    The  single  most  important  
agent  of  Traveler's  diarrhea  

 A.  Enterotoxigenic  E.  coli  


 B.  Salmonella  typhi  
 C.  Campylobacter  jejuni  
 D.  Rotavirus  
 
154.  Least  common  but  the  most  
severe  of  the  pneumonic  
complicaLons  of  influenza  
A.  Secondary  bacterial  pneumonia  
B.  Primary  influenza  viral  pneumonia  
C.  Mixed  viral  and  bacterial  pneumonia  
D.  Chronic  bronchiLs  with  aspiraLon  
pneumonia  
154.  Least  common  but  the  most  
severe  of  the  pneumonic  
complicaLons  of  influenza  
A.  Secondary  bacterial  pneumonia  
B.  Primary  influenza  viral  pneumonia  
C.  Mixed  viral  and  bacterial  pneumonia  
D.  Chronic  bronchiLs  with  aspiraLon  
pneumonia  
Influenza-­‐related  Pneumonia  
TYPE   CLINICAL  SIGNS/SYMPTOMS   REMARKS  
Primary  influenza   Acute  influenza  that  does  not   At  risk:  MS,  pregnancy,  
pneumonia   resolve  but  instead  progresses   elderly,  COPD  
relentlessly,  with  persistent    
fever,  dyspnea,  and  eventual   CXR:  IntersLLal  infiltrates  
cyanosis   ARDS-­‐like  features  
Secondary   Improvement  of  the  paLent's   At  risk:  COPD,  elderly  
bacterial   condiLon  over  2–3  days  is    
pneumonia   followed  by  a  reappearance  of   Agents:  Streptococcus  
fever  along  with  clinical  signs   pneumoniae,  
and  symptoms  of  bacterial   Staphylococcus  aureus,  
pneumonia   Haemophilus  influenzae  
Mixed  Viral  and   Mixed  features  of  viral  and   At  risk:  COPD,  cardiac  
Bacterial   bacterial  pneumonia   diseases  
Pneumonia    
CXR:  Patchy  infiltrates  or  
areas  of  consolidaLon    
155.  Which  Plasmodium  species  has  
72-­‐hour  duraLon  of  erythrocyLc  stage,  
15-­‐day  duraLon  of  intrahepaLc  phase  
and  preference  for  older  red  cells  

A.  P.  vivax    
B.  P.  ovale        
C.  P.  malariae    
D.  P.  falciparum    
155.  Which  Plasmodium  species  has  
72-­‐hour  duraLon  of  erythrocyLc  stage,  
15-­‐day  duraLon  of  intrahepaLc  phase  
and  preference  for  older  red  cells  

A.  P.  vivax    
B.  P.  ovale        
C.  P.  malariae    
D.  P.  falciparum    
Malaria  
156.  A  30  year  old  male  comes  in  for  
agitaLon  and  hydrophobia.    On  further  
probing,  his  relaLves  recall  a  history  of  
dog  bite  2  weeks  prior.    The  most  
characterisLc  pathologic  finding  in  this  
disease  is:  
A.  Negri  body    
B.  Amyloid  body    
C.  Babes  nodules    
D.  Filamentous  virion    
156.  A  30  year  old  male  comes  in  for  
agitaLon  and  hydrophobia.    On  further  
probing,  his  relaLves  recall  a  history  of  
dog  bite  2  weeks  prior.    The  most  
characterisLc  pathologic  finding  in  this  
disease  is:  
A.  Negri  body    
B.  Amyloid  body    
C.  Babes  nodules    
D.  Filamentous  virion    
157.  Largest  intesLnal  nematode  
parasite  of  humans,  reaching  up  to  
40cms  in  length    

A. Trichuris  trichiura    
B. Ascaris  lumbricoides    
C. Stongyloides  stercoralis    
D. Enterobius  vermicularis    
157.  Largest  intesLnal  nematode  
parasite  of  humans,  reaching  up  to  
40cms  in  length    

A. Trichuris  trichiura    
B. Ascaris  lumbricoides    
C. Stongyloides  stercoralis    
D. Enterobius  vermicularis    
158.  A  30  year  old  commercial  sex  worker  
comes  in  for  recurrent  whiLsh  material  on  
her  tongue.    She  has  had  weight  loss,  daily  
febrile  episodes,  and  generalized  
lymphadenopathies.    To  confirm  your  
diagnosis,  you  will  request  for:  

A.  ELISA    
B.  RNA  PCR    
C.  Western  blot    
D.  p24  anLgen  capture    
158.  A  30  year  old  commercial  sex  worker  
comes  in  for  recurrent  whiLsh  material  on  
her  tongue.    She  has  had  weight  loss,  daily  
febrile  episodes,  and  generalized  
lymphadenopathies.    To  confirm  your  
diagnosis,  you  will  request  for:  

A.  ELISA    
B.  RNA  PCR    
C.  Western  blot    
D.  p24  anLgen  capture    
159.    Mainstay  preferred  
diagnosLc  test  for  Leptospirosis  

A.  Culture  assays  


B.  Serologic  tests  
C.  PCR  idenLficaLon  
D.  Dark-­‐field  microscopy  
159.    Mainstay  preferred  
diagnosLc  test  for  Leptospirosis  

A.  Culture  assays  


B.  Serologic  tests  
C.  PCR  idenLficaLon  
D.  Dark-­‐field  microscopy  
160.    CondiLon  with  increased  
suscepLbility  to  Salmonella  
infecLons  
A.    Alcoholism  
B      Antacid  ingesLon  
C.    Age  less  than  5  years  
D.    Maternal  colonizaLon  
 
160.    CondiLon  with  increased  
suscepLbility  to  Salmonella  
infecLons  
A.    Alcoholism  
B      Antacid  ingesNon  
C.    Age  less  than  5  years  
D.    Maternal  colonizaLon  
 
Increased  Risk  for  Salmonella:  
•  Decreased  gastric  acidity  
–  Antacids  
–  Achlorhydria  
 
•  GI  mucosal  alteraLon  
–  Prior  surgery  
–  Inflammatory  bowel  disease  
161.    Best  diagnosLc  test  for  
localized  and  generalized  
tetanus  infecLon  
     
A.  Wound  culture    
B.  Tetanus  toxin  PCR  
C.  Clostridium  IgM  
D.  None  needed  
 
161.    Best  diagnosLc  test  for  
localized  and  generalized  
tetanus  infecLon  
     
A.  Wound  culture    
B.  Tetanus  toxin  PCR  
C.  Clostridium  IgM  
D.  None  needed  
 
162.  Preferred  anLbioLc  for  
tetanus  infecLon  
A.    CeWriaxone    
B.    Penicillin  G  
C.    Clindamycin    
D.    Metronidazole  
 
162.  Preferred  anLbioLc  for  
tetanus  infecLon  
A.    CeWriaxone    
B.    Penicillin  G  
C.    Clindamycin    
D.    Metronidazole  
 
163.    InfecLon  predominantly  
transmised  by  sexual  
intercourse  
A.  HepaLLs  C  virus  
B. Sarcoptes  scabiae  
C. Chlamydia  trachoma-s  
D. Gardnerella  vaginalis  
163.    InfecLon  predominantly  
transmised  by  sexual  
intercourse  
A.  HepaLLs  C  virus  
B. Sarcoptes  scabiae  
C. Chlamydia  trachoma8s  
D. Gardnerella  vaginalis  
164.    Most  common  cause  of  acute  
epididymiLs  in  sexually  acLve  men  
less  than  35  years  old  
 
A.  Chlamydia  trachomaLs  
B.  Neisseria  gonorrhoea  
C.  Enterobacteriaceae  
D.  Herpes  simplex  virus  type  2  
164.    Most  common  cause  of  acute  
epididymiLs  in  sexually  acLve  men  
less  than  35  years  old  
 
A.  Chlamydia  trachomaNs  
B.  Neisseria  gonorrhoea  
C.  Enterobacteriaceae  
D.  Herpes  simplex  virus  type  2  
165.    Most  common  manifestaLon  of  
infecLon  with  Clostridium  difficile  
A. Fever    
B. Diarrhea  
C. Abdominal  pain  
D. Leucocytosis  
165.    Most  common  manifestaLon  of  
infecLon  with  Clostridium  difficile  
A. Fever    
B.  Diarrhea  
C. Abdominal  pain  
D. Leucocytosis  
166.    What  is  the  normal  mean  annual  
decline  in  glomerular  filtraLon  rate  
with  age  in  ml/min/body  surface  area?  

A.  0.5  
B.  1.0  
C.  1.5  
D.  2.0  
 
166.    What  is  the  normal  mean  annual  
decline  in  glomerular  filtraLon  rate  
with  age  in  ml/min/body  surface  area?  

A.  0.5  
B.  1.0  
C.  1.5  
D.  2.0  
 
167.    Which  of  the  following  
suggest  a  chronic  eLology  for  the  
kidney  disease?  

A.  Bilateral  kidney  size  on  ultrasound  10  cm  


B.  ElevaLon  of  serum  creaLnine  >6  weeks  
C.  OsteiLs  fibrosa  cysLca  
D.  Presence  of  oliguria  
 
167.    Which  of  the  following  
suggest  a  chronic  eLology  for  the  
kidney  disease?  

A.  Bilateral  kidney  size  on  ultrasound  10  cm  


B.  ElevaLon  of  serum  creaLnine  >6  weeks  
C.  OsteiNs  fibrosa  cysNca  
D.  Presence  of  oliguria  
 
168.  This  diureLc  reversible  inhibit  
the  resorpLon  of  Na-­‐K-­‐Cl  in  the  
thick  ascending  limb  of  Henle’s  
loop  
A. Bumetanide  
B. Thiazide  
C. Metolazone  
D. Spironolactone  
168.  This  diureLc  reversible  inhibit  
the  resorpLon  of  Na-­‐K-­‐Cl  in  the  
thick  ascending  limb  of  Henle’s  
loop  
A. Bumetanide  
B. Thiazide  
C. Metolazone  
D. Spironolactone  
169.  Which  of  the  following  
diureLcs  can  retain  potassium?  
A.  Furosemide  
B.  Chlorthalidone  
C.  Triamterene  
D.  Ethacrynic  acid  
169.  Which  of  the  following  
diureLcs  can  retain  potassium?  
A.  Furosemide  
B.  Chlorthalidone  
C.  Triamterene  
D.  Ethacrynic  acid  
170.  The  pentad  of  thromboLc  
thrombocytopenic  purpura  
includes  the  following,  EXCEPT  

A.  HemolyLc  anemia  


B.  Thrombocytopenia  
C.  Neurologic  symptoms  
D.  HepaLc  failure  
 
170.  The  pentad  of  thromboLc  
thrombocytopenic  purpura  
includes  the  following,  EXCEPT  

A.  HemolyLc  anemia  


B.  Thrombocytopenia  
C.  Neurologic  symptoms  
D.  HepaNc  failure  
 
TTP  
•  PENTAD  of  findings:  
–  Microangiopathic  hemolyLc  anemia  
–  Thrombocytopenia  
–  Renal  failure    
–  Neurologic  findings  
–  Fever  
171.  Primary  renal  pathophysiologic  
lesion  in  acute  leptospirosis  is  injury  
in  the  _______________  
 
A.  Glomerulus  
B.  Ascending  loop  of  henle    
C.  Descending  loop  of  henle    
D.  Proximal  convoluted  tubule    
171.  Primary  renal  pathophysiologic  
lesion  in  acute  leptospirosis  is  injury  
in  the  _______________  
 
A.  Glomerulus  
B.  Ascending  loop  of  henle    
C.  Descending  loop  of  henle    
D.  Proximal  convoluted  tubule    
172.  Most  common  cause  of  
nephroLc  syndrome  in  the  elderly  

A.  Minimal  change  disease    


B.  Membranous  glomerulonephriLs  
C.  Focal  segmental  glomerulosclerosis  
D.  MembranoproliferaLve  
glomerulonephriLs  
172.  Most  common  cause  of  
nephroLc  syndrome  in  the  elderly  

A.  Minimal  change  disease    


B.  Membranous  glomerulonephriNs  
C.  Focal  segmental  glomerulosclerosis  
D.  MembranoproliferaLve  
glomerulonephriLs  
173.    KimmelsLel-­‐Wilson  
nodules  are  seen  in:    
A.  Fabry’s  disease    
B.  Urate  nephropathy  
C.  DiabeLc  Nephropathy    
D.  Analgesic  nephropathy  
173.    KimmelsLel-­‐Wilson  
nodules  are  seen  in:    
A.  Fabry’s  disease    
B.  Urate  nephropathy  
C.  DiabeNc  Nephropathy    
D.  Analgesic  nephropathy  
174.  First  line  of  therapy  for  the  
management  of  hypertension  in  
CKD    
A. Salt  restricLon    
B. ACE  inhibitors    
C. Loop  diureLcs    
D. Calcium  channel  blockers    
174.  First  line  of  therapy  for  the  
management  of  hypertension  in  
CKD    
A. Salt  restricNon    
B. ACE  inhibitors    
C. Loop  diureLcs    
D. Calcium  channel  blockers    
175.  Diagnosis  for  thyroid  biopsy  
revealing  psammoma  bodies  with  
Orphan  Annie  appearance  

A.  AnaplasLc  thyroid  cancer  


B.  Follicular  thyroid  cancer  
C.  Papillary  thyroid  cancer  
D.  Thyroid  lymphoma  
 
175.  Diagnosis  for  thyroid  biopsy  
revealing  psammoma  bodies  with  
Orphan  Annie  appearance  

A.  AnaplasLc  thyroid  cancer  


B.  Follicular  thyroid  cancer  
C.  Papillary  thyroid  cancer  
D.  Thyroid  lymphoma  
 
176.     A  55  year  old  diabeLc  female  came  in  for  
decrease  in  sensorium.  
   CBG  showed  780mg/dL    
 ABG    pH  of  7.32  HCO3  20  pO2  92  pCO2  30    
 On  further  inquiring,  she  was  poorly  
compliant  to  her  medicaLons  and  was  
complaining  of  cough  with  yellowish  sputum  
for  3  days,  vomiLng  abdominal  pain  for  1  
day.  There  was  glucosuria  and  +1  ketonuria  
on  urinalysis.  
What  is  the  most  likely  
diagnosis?  
A.  DiabeLc  ketoacidosis  
B.  Hyperglycemic  hyperosmolar  state  
C.  Cerebral  infarct  
D.  Sepsis  syndrome  
 
What  is  the  most  likely  
diagnosis?  
A.  DiabeLc  ketoacidosis  
B.  Hyperglycemic  hyperosmolar  state  
C.  Cerebral  infarct  
D.  Sepsis  syndrome  
 
DKA  VS  HHS  
177.  What  is  an  appropriate  
iniLal  resuscitaLon  step?  
A.  Give  insulin  IV  bolus  with  potassium  
B.  Hydrate  with  plain  NSS  
C.  Order  for  STAT  plain  cranial  CT  
D.  Start  broad-­‐spectrum  IV  anLbioLcs  
 
177.  What  is  an  appropriate  
iniLal  resuscitaLon  step?  
A.  Give  insulin  IV  bolus  with  potassium  
B.  Hydrate  with  plain  NSS  
C.  Order  for  STAT  plain  cranial  CT  
D.  Start  broad-­‐spectrum  IV  anLbioLcs  
 
178.    What  BP  level  is  
considered  hypertension  in  
diabeLc  paLents?  
A.  >  120/80  
B.  >  125/75  
C.  >  130/80  
D.  >  140/90  
 
178.    What  BP  level  is  
considered  hypertension  in  
diabeLc  paLents?  
A.  >  120/80  
B.  >  125/75  
C.  >  130/80  
D.  >  140/90  
 
179.    Which  of  the  following  
insulin  preparaLon  provide  
basal  insulin?  
A.  NPH  insulin  
B.  Insulin  aspart    
C.  Regular  insulin    
D.  Insulin  glargine  
 
179.    Which  of  the  following  
insulin  preparaLon  provide  
basal  insulin?  
A.  NPH  insulin  
B.  Insulin  aspart    
C.  Regular  insulin    
D.  Insulin  glargine  
 
180.    Diabetes  medicaLons  that  
can  cause  hypoglycemia  even  if  
used  alone  
 A.  Me•ormin  
 B.  Alpha-­‐glucosidase  inhibitors  
 C.  Thiazolidinediones  
 D.  Sulfonylureas  
 
180.    Diabetes  medicaLons  that  
can  cause  hypoglycemia  even  if  
used  alone  
 A.  Me•ormin  
 B.  Alpha-­‐glucosidase  inhibitors  
 C.  Thiazolidinediones  
 D.  Sulfonylureas  
 
181.  Microadenomas  differ  
from  macroadenomas  in  size.    
What  is  the  cut  off?  
A. 5  mm  
B. 10  mm  
C. 15  mm  
D. 20mm  
181.  Microadenomas  differ  
from  macroadenomas  in  size.    
What  is  the  cut  off?  
A. 5  mm  
B.  10  mm  
C. 15  mm  
D. 20mm  
182.  A  35  year  old  female  presents  with  
amenorrhea,  inferLlity,  and  galactorrhea.    
ProlacLn  level  was  significantly  elevated  at  
>  200  g/L.    An  MRI  revealed  findings  
suggesLve  of  microadenoma.    The  
mainstay  of  therapy  is:  

A.  Dopamine  agonist    


B.  Surgical  debulking  
C.  Transphenoid  resecLon  
D.  Radiotherapy  
182.  A  35  year  old  female  presents  with  
amenorrhea,  inferLlity,  and  galactorrhea.    
ProlacLn  level  was  significantly  elevated  at  
>  200  g/L.    An  MRI  revealed  findings  
suggesLve  of  microadenoma.    The  
mainstay  of  therapy  is:  

A.  Dopamine  agonist    


B.  Surgical  debulking  
C.  Transphenoid  resecLon  
D.  Radiotherapy  
183.  A  20  year  old  male  was  found  to  
have  a  parathyroid  adenoma,  pituitary  
adenoma,  and  pancreaLc  tumor.    This  
familial  pituitary  syndrome  is  called:  

A.  MEN-­‐1  
B.  MEN-­‐2A  
C.  MEN-­‐2B  
D.  MEN-­‐3  
183.  A  20  year  old  male  was  found  to  
have  a  parathyroid  adenoma,  pituitary  
adenoma,  and  pancreaLc  tumor.    This  
familial  pituitary  syndrome  is  called:  

A.  MEN-­‐1  
B.  MEN-­‐2A  
C.  MEN-­‐2B  
D.  MEN-­‐3  
Disease  AssociaLons  in  the  MulLple  Endocrine  
Neoplasia  (MEN)  Syndromes  
 
MEN  1   MEN  2A   MEN  2B  
Parathyroid  hyperplasia  or          Medullary  Thyroid        Medullary  Thyroid  
adenoma   Carcinoma   Carcinoma  
     
Islet  cell  hyperplasia,          Pheochromocytoma        Pheochromocytoma  
adenoma,  or  carcinoma                  
  Parathyroid  hyperplasia   Mucosal  and  
Pituitary  hyperplasia  or   or  adenoma   gastrointesLnal  
adenoma     neuromas  
   
Other,  less  common   Marfanoid  features  
manifestaLons:  foregut    
carcinoid,  
pheochromocytoma,  
subcutaneous  or  visceral  
lipomas  
 
184.  A  46  year  old  female  has  a  3cm  
thyroid  nodule  at  the  isthmus.    Which  
feature  is  a  risk  factor  for  thyroid  
carcinoma  for  this  paLent?  

A. Age  
B. Sex  
C. Nodule  size  
D. Nodule  locaLon  
184.  A  46  year  old  female  has  a  3cm  
thyroid  nodule  at  the  isthmus.    Which  
feature  is  a  risk  factor  for  thyroid  
carcinoma  for  this  paLent?  

A. Age  
B. Sex  
C. Nodule  size  
D. Nodule  locaLon  
Risk  Factors  for  Thyroid  Carcinoma  in  
PaLents  with  Thyroid  Nodule  
•  History  of  head  and  neck  irradiaLon    
•  Age  <20  or  >45  years  
•   Bilateral  disease    
•  Increased  nodule  size  (>4  cm)    
•  New  or  enlarging  neck  mass    
•  Male  gender  
•   Family  history  of  thyroid  cancer  or  MEN  2    
•  Vocal  cord  paralysis,  hoarse  voice    
•  Nodule  fixed  to  adjacent  structures    
•  Extrathyroidal  extension    
•  Suspected  lymph  node  involvement    
•  Iodine  deficiency  (follicular  cancer)  
185.  A  35  year  old  female  with  
palpitaLons  has  a  low  TSH  and  a  
normal  FT4.    Your  next  approach  to  
management:  
A.  Treat  as  Graves’  disease  with  methimazole  
B.  Get  an  MRI  to  rule  out  a  TSH  secreLng  
pituitary  adenoma  
C.  Treat  as  subclinical  hyperthyroidism  and  
observe    
D.  Measure  FT3  to  rule  out  T3  toxicosis  
185.  A  35  year  old  female  with  
palpitaLons  has  a  low  TSH  and  a  
normal  FT4.    Your  next  approach  to  
management:  
A.  Treat  as  Graves’  disease  with  methimazole  
B.  Get  an  MRI  to  rule  out  a  TSH  secreLng  
pituitary  adenoma  
C.  Treat  as  subclinical  hyperthyroidism  and  
observe    
D.  Measure  FT3  to  rule  out  T3  toxicosis  
186.    A  28  year  old  pregnant  female  
was  diagnosed  with  Graves’  disease.    
You  prefer  propylthiouracil  for  this  
paLent  because:  

A.  Has  beser  bioavailability  during  pregnancy  


B.  It  has  low  transplacental  transfer  
C.  It  is  more  effecLve  than  methimazole  in  
pregnant  paLents  
D.  Methimazole  does  not  cross  the  placenta  
during  pregnancy  
186.    A  28  year  old  pregnant  female  
was  diagnosed  with  Graves  disease.    
You  prefer  propylthiouracil  for  this  
paLent  because:  

A.  Has  beser  bioavailability  during  pregnancy  


B.  It  has  low  transplacental  transfer  
C.  It  is  more  effecLve  than  methimazole  in  
pregnant  paLents  
D.  Methimazole  does  not  cross  the  placenta  
during  pregnancy  
187.  Screening  for  diabetes  
mellitus  is  recommended  for  all  
individuals  who  are  more  than:  
A. 25  years  old  
B. 35  years  old  
C. 45  years  old  
D. 55  years  old  
187.  Screening  for  diabetes  
mellitus  is  recommended  for  all  
individuals  who  are  more  than:  
A. 25  years  old  
B. 35  years  old  
C.  45  years  old  
D. 55  years  old  
188.    Which  of  the  following  
finding  is  consistent  with  portal  
hypertension?  
A.  Hypoalbuminemia  
B.  Obliterated  Traube’s  space  
C.  Spider  angioma  
D.  AST:  ALT  raLo  more  than  2  
 
188.    Which  of  the  following  
finding  is  consistent  with  portal  
hypertension?  
A.  Hypoalbuminemia  
B.  Obliterated  Traube’s  space  
C.  Spider  angioma  
D.  AST:  ALT  raLo  more  than  2  
 
189.    What  is  the  role  of  lactulose  
in  paLents  with  hepaLc  
encephalopathy?  
A.  Evacuate  blood  from  the  
gastrointesLnal  tract  
B.  InacLvate  colonic  bacteria  
C.  Induce  diarrhea    
D.  Promote  colonic  alkalinizaLon  
 
189.    What  is  the  role  of  lactulose  
in  paLents  with  hepaLc  
encephalopathy?  
A.  Evacuate  blood  from  the  
gastrointesLnal  tract  
B.  InacNvate  colonic  bacteria  
C.  Induce  diarrhea    
D.  Promote  colonic  alkalinizaLon  
 
190.    Associated  with  increased  
risk  of  pepLc  ulcer  disease:  

A.  Atopic  diseases  
B.  Nephrolithiases  
C.  Blood  group  A  
D.  Acute  renal  failure  
 
190.    Associated  with  increased  
risk  of  pepLc  ulcer  disease:  

A.  Atopic  diseases  
B.  Nephrolithiases  
C.  Blood  group  A  
D.  Acute  renal  failure  
 
Increased  Risk  for  PUD  
•  Strong  associaLon   •  Possible  associaLon    
–  Systemic  mastocytosis   –  Hyperparathyroidism,  
–  Chronic  pulmonary   coronary  artery  disease,  
disease   polycythemia  vera,  
–  Chronic  renal  failure   chronic  pancreaLLs  
–  Cirrhosis   •  Blood  type  O  
–  Nephrolithiasis   •  NSAIDs  
–  Alpha  1-­‐anLtrypsin  
deficiency  
191.  Least  common  
complicaLon  of  pepLc  ulcer  

A. Gastric  outlet  obstrucLon  


B. PerforaLon  
C. Bleeding  
D. GastriLs  
191.  Least  common  
complicaLon  of  pepLc  ulcer  

A. Gastric  outlet  obstrucNon  


B. PerforaLon  
C. Bleeding  
D. GastriLs  
192.  A  20  year  old  female  comes  in  for  
abdominal  pain  occurring  3  days  per  
month  in  the  last  three  months.    
Associated  symptoms  include  
improvement  of  pain  with  defecaLon  and  
soW  frequent  stools.    She  most  likely  has:  

A.  Irritable  bowel  syndrome  


B.  Inflammatory  bowel  disease  
C.  GastrointesLnal  tuberculosis  
D.  IntesLnal  parasiLsm  
192.  A  20  year  old  female  comes  in  for  
abdominal  pain  occurring  3  days  per  
month  in  the  last  three  months.    
Associated  symptoms  include  
improvement  of  pain  with  defecaLon  and  
soW  frequent  stools.    She  most  likely  has:  

A.  Irritable  bowel  syndrome  


B.  Inflammatory  bowel  disease  
C.  GastrointesLnal  tuberculosis  
D.  IntesLnal  parasiLsm  
193.  A  50  year  old  male,  smoker,  alcoholic  
drinker  comes  in  for  sudden  severe  
nonremizng  abdominal  pain.    Heart  rate  was  
noted  to  be  irregularly  irregular.    There  was  only  
very  minimal  tenderness,  out  of  proporLon  to  
the  symptoms.    The  gold  standard  of  diagnosis  
is:  

A.  Radiograph  showing  bowel-­‐wall  edema  


(“thumbprinLng”)  
B.  DemonstraLon  of  pneumatosis  intesLnalis  
C.  Oral  and  IV  dynamic  CT  scan  
D.  Lapatoromy  
193.  A  50  year  old  male,  smoker,  alcoholic  
drinker  comes  in  for  sudden  severe  
nonremizng  abdominal  pain.    Heart  rate  was  
noted  to  be  irregularly  irregular.    There  was  only  
very  minimal  tenderness,  out  of  proporLon  to  
the  symptoms.    The  gold  standard  of  diagnosis  
is:  

A.  Radiograph  showing  bowel-­‐wall  edema  


(“thumbprinLng”)  
B.  DemonstraLon  of  pneumatosis  intesLnalis  
C.  Oral  and  IV  dynamic  CT  scan  
D.  Lapatoromy  
194.  Single  most  common  risk  
factor  for  hepaLLs  C  
A. HIV  
B. Blood  transfusion  
C. InjecLon  drug  use  
D. Risky  sexual  behavior  
194.  Single  most  common  risk  
factor  for  hepaLLs  C  
A. HIV  
B. Blood  transfusion  
C.  InjecNon  drug  use  
D. Risky  sexual  behavior  
195.  Best  therapy  for  
hepatorenal  syndrome  

A. Dialysis  
B. OctreoLde    
C. Midodrine    
D. Liver  transplant    
195.  Best  therapy  for  
hepatorenal  syndrome  

A. Dialysis  
B. OctreoLde    
C. Midodrine    
D. Liver  transplant    
196.        Most  common  cause  of  
rectal  bleeding  in  infancy  
 A.  Trauma    
 B.  Anal  fissure  
 C.  Hemorrhoids  
 D.  Imperforate  anus  
 
196.        Most  common  cause  of  
rectal  bleeding  in  infancy  
 A.  Trauma    
 B.  Anal  fissure  
 C.  Hemorrhoids  
 D.  Imperforate  anus  
 
197.    Best  imaging  study  for  iniLal  
evaluaLon  of  a  suspected  pancreaLc  
disorder  and  for  the  complicaLons  of  
acute  and  chronic  pancreaLLs  

 A.  CT    
 B.  MRI  
 C.  ERCP  
 D.  Ultrasound  
 
197.    Best  imaging  study  for  iniLal  
evaluaLon  of  a  suspected  pancreaLc  
disorder  and  for  the  complicaLons  of  
acute  and  chronic  pancreaLLs  

 A.  CT    
 B.  MRI  
 C.  ERCP  
 D.  Ultrasound  
 
198.    Leading  cause  of  acute  
pancreaLLs  
 A.  Idiopathic  
 B.  Alcoholism  
 C.  Cholelithiases  
 D.  Hypertriglyceridemia  
 
198.    Leading  cause  of  acute  
pancreaLLs  
 A.  Idiopathic  
 B.  Alcoholism  
 C.  Cholelithiases  
 D.  Hypertriglyceridemia  
 
199.    Which  of  the  following  lab  tests  
used  in  the  diagnosis  of  acute  
pancreaLLs  remain  elevated  for  7  to  
14  days  aWer  the  iniLal  bout?  

 A.  Lipase  
 B.  Calcium  
 C.  Amylase  
 D.  Hematocrit  
 
199.    Which  of  the  following  lab  tests  
used  in  the  diagnosis  of  acute  
pancreaLLs  remain  elevated  for  7  to  
14  days  aWer  the  iniLal  bout?  

 A.  Lipase  
 B.  Calcium  
 C.  Amylase  
 D.  Hematocrit  
 
200.  Hallmark  of  anaphylaxis    
A.  Vascular  collapse  leading  to  
hypotension    
B.  UrLcarial  erupLons  which  are  intensely  
pruriLc    
C.  Respiratory  distress  from  laryngeal  
edema    
D.  Rapid  onset  aWer  introducLon  of  
anLgen    
200.  Hallmark  of  anaphylaxis    
A.  Vascular  collapse  leading  to  
hypotension    
B.  UrLcarial  erupLons  which  are  intensely  
pruriLc    
C.  Respiratory  distress  from  laryngeal  
edema    
D.  Rapid  onset  aber  introducNon  of  
anNgen    
201.  Chronic  UrLcaria  is  defined  
as  asacks  lasLng  more  than  

A. 2  weeks    
B. 4  weeks    
C. 6  weeks    
D. 8  weeks    
201.  Chronic  UrLcaria  is  defined  
as  asacks  lasLng  more  than  

A. 2  weeks    
B. 4  weeks    
C.  6  weeks    
D. 8  weeks    
202.    Key  effector  cell  of  the  
biologic  response  in  allergic  
rhiniLs,  urLcaria,  anaphylaxis  and  
systemic  mastocytosis  
A. Mast  cell  
B. Basophil    
C. Eosinophil  
D. DendriLc  cell  
202.    Key  effector  cell  of  the  
biologic  response  in  allergic  
rhiniLs,  urLcaria,  anaphylaxis  and  
systemic  mastocytosis  
A. Mast  cell  
B. Basophil    
C. Eosinophil  
D. DendriLc  cell  
203.  A  20  year  old  paLent  develops  
severe  maculopapular  rashes  all  over  
aWer  a  bee  sLng.    Associated  signs  and  
symptoms  include  dyspnea,  wheezing,  
vomiLng,  an  dizziness.  Drug  of  choice  is:      

A.  Diphenhydramine  
B.  HydrocorLsone  
C.  Norepinephrine  
D.  Epinephrine  
203.  A  20  year  old  paLent  develops  
severe  maculopapular  rashes  all  over  
aWer  a  bee  sLng.    Associated  signs  and  
symptoms  include  dyspnea,  wheezing,  
vomiLng,  an  dizziness.  Drug  of  choice  is:      

A.  Diphenhydramine  
B.  HydrocorLsone  
C.  Norepinephrine  
D.  Epinephrine  
204.  Most  common  pulmonary  
manifestaLon  of  SLE  
A. PleuriLs    
B. Pneumonia    
C. Pleural  effusion    
D. Pulmonary  fibrosis    
204.  Most  common  pulmonary  
manifestaLon  of  SLE  
A. PleuriNs    
B. Pneumonia    
C. Pleural  effusion    
D. Pulmonary  fibrosis    
205.  “Salt  and  pepper”  
appearance  of  the  skin  is  seen  in:  

A.  Systemic  Lupus  Erythematosus    


B.  Systemic  Sclerosis    
C.  Ankylosing  SpondyliLs    
D.  Behcet’s  Syndrome    
205.  “Salt  and  pepper”  
appearance  of  the  skin  is  seen  in:  

A.  Systemic  Lupus  Erythematosus    


B.  Systemic  Sclerosis    
C.  Ankylosing  SpondyliLs    
D.  Behcet’s  Syndrome    
206.  The  pathergy  test  is  used  in  
the  diagnosis  of:    
A.  Henoch  Schonlein  Purpura    
B.  Takayasu  ArteriLs    
C.  Behcet’s  Syndrome    
D.  Ankylosing  SpondyliLs    
206.  The  pathergy  test  is  used  in  
the  diagnosis  of:    
A.  Henoch  Schonlein  Purpura    
B.  Takayasu  ArteriLs    
C.  Behcet’s  Syndrome    
D.  Ankylosing  SpondyliLs    
DiagnosLc  Criteria  for  Behcet’s  
–  Recurrent  Oral  UlceraNon  plus  TWO  of  the  
following:  
•  Recurrent  Genital  UlceraLon  
•  Eye  lesions  
•  Skin  lesions  
•  Pathergy  Test  (Nonspecific  skin  inflammatory  reacLvity  to  any  scratches  or  
intradermal  saline  injecLon)  
207.  Accounts  for  most  non-­‐
gonococcal  isolates  in  infecLous  
arthriLs  in  adults  of  all  ages    

A.  Staphylococcus  aureus    


B.  Gram  negaLve  bacilli    
C.  Β-­‐hemolyLc  streptococci    
D.  Mycobacterium  tuberculosis  
207.  Accounts  for  most  non-­‐
gonococcal  isolates  in  infecLous  
arthriLs  in  adults  of  all  ages    

A.  Staphylococcus  aureus    


B.  Gram  negaLve  bacilli    
C.  Β-­‐hemolyLc  streptococci    
D.  Mycobacterium  tuberculosis  
208.    Most  common  variety  of  
psoriasis  
A. Gusate  
B. Pustular  
C. Plaque-­‐type  
D. Inverse  
208.    Most  common  variety  of  
psoriasis  
A. Gusate  
B. Pustular  
C.  Plaque-­‐type  
D. Inverse  
209.  Most  common  
dermatophyte  infecLon    
A.  Tinea  corporis    
B.  Tinea  cruris    
C.  Tinea  capiLs    
D.  Tinea  pedis    
209.  Most  common  
dermatophyte  infecLon    
A.  Tinea  corporis    
B.  Tinea  cruris    
C.  Tinea  capiLs    
D.  Tinea  pedis    
210.    This  differenLates  primary  
from  secondary  causes  of  
polycythemia  

A. RBC  mass  
B. EPO  levels  
C. Arterial  O2  saturaLon  
D. Carboxyhemoglobin  levels  
210.    This  differenLates  primary  
from  secondary  causes  of  
polycythemia  

A. RBC  mass  
B.  EPO  levels  
C. Arterial  O2  saturaLon  
D. Carboxyhemoglobin  levels  
•  Low  EPO  à  Polycythemia  Vera  
•  High  EPO  à  Secondary  Causes  
211.    Most  convenient  
laboratory  test  to  diagnose  iron  
deficiency  anemia  

A. TIBC  
B. Serum  iron  
C. Serum  ferriLn  
D. Serum  transferrin    
211.    Most  convenient  
laboratory  test  to  diagnose  iron  
deficiency  anemia  

A. TIBC  
B. Serum  iron  
C.  Serum  ferriNn  
D. Serum  transferrin    
212.  Thalassemia  might  favor  a  
natural  protecLon  against  which  
infecLon?  
A.  Malaria    
B.  Dengue  Fever    
C.  Typhoid  Fever    
D.  HIV    
212.  Thalassemia  might  favor  a  
natural  protecLon  against  which  
infecLon?  
A.  Malaria    
B.  Dengue  Fever    
C.  Typhoid  Fever    
D.  HIV    
213.  Mainstay  of  treatment  for  
sickle  cell  anemia  
A.  HydraLon    
B.  Hydroxyurea    
C.  Red  Cell  Transfusion    
D.  Bone  marrow  transplant    
213.  Mainstay  of  treatment  for  
sickle  cell  anemia  
A.  HydraLon    
B.  Hydroxyurea    
C.  Red  Cell  Transfusion    
D.  Bone  marrow  transplant    
214.    OsmoLc  Fragility  Test  
is  used  to  diagnose:  
A. G6PD  deficiency    
B. Sickle  cell  anemia  
C. Hereditary  spherocytosis    
D. Paroxysmal  nocturnal  
hemoglobinuria    
214.    OsmoLc  Fragility  Test  
is  used  to  diagnose:  
A. G6PD  deficiency    
B. Sickle  cell  anemia  
C.  Hereditary  spherocytosis    
D. Paroxysmal  nocturnal  
hemoglobinuria    
215.  Most  common  physical  
finding  in  Chronic  Myelogenous  
Leukemia  
 
A. Pallor  
B. Petechiae  
C. Lymphadenopathy  
D. Splenomegaly  
215.  Most  common  physical  
finding  in  Chronic  Myelogenous  
Leukemia  
 
A. Pallor  
B. Petechiae  
C. Lymphadenopathy  
D. Splenomegaly  
216.  A  50  year  old  female  diagnosed  
with  leukemia  showed  “smudge”  or  
basket  cells  on  peripheral  blood  
smear.    Your  diagnosis  is:  

A.  Acute  promyelocyLc  leukemia  


B.  Chronic  lymphocyLc  leukemia  
C.  Acute  lymphoblasLc  leukemia  
D.  Mantle  cell  lymphoma  
216.  A  50  year  old  female  diagnosed  
with  leukemia  showed  “smudge”  or  
basket  cells  on  peripheral  blood  
smear.    Your  diagnosis  is:  

A.  Acute  promyelocyLc  leukemia  


B.  Chronic  lymphocyNc  leukemia  
C.  Acute  lymphoblasLc  leukemia  
D.  Mantle  cell  lymphoma  
217.  Most  rapidly  progressive  
lymphoma  
A.  Diffuse  large  B-­‐cell  lymphoma  
B.  Burkis’s  lymphoma  
C.  Mantle  cell  lymphoma    
D.  Follicular  lymphoma  
217.  Most  rapidly  progressive  
lymphoma  
A.  Diffuse  large  B-­‐cell  lymphoma  
B.  Burkir’s  lymphoma  
C.  Mantle  cell  lymphoma    
D.  Follicular  lymphoma  
218.  Given  with  Cyclophosphamide  
chemotherapy  to  reduce  the  risk  of  
bladder  damage    
A. N-­‐acetylcysteine    
B. Mesna    
C. Acrolein    
D. Leucovorin    
218.  Given  with  Cyclophosphamide  
chemotherapy  to  reduce  the  risk  of  
bladder  damage    
A. N-­‐acetylcysteine    
B.  Mesna    
C. Acrolein    
D. Leucovorin    
219.  Co-­‐administered  with  5-­‐
fluorouracil  to  enhance  
cytotoxicity  in  the  treatment  of  GI  
cancers  
A.  N-­‐acetylcysteine  
 
B.  Mesna  
 
C.  Acrolein  
D. Leucovorin  
219.  Co-­‐administered  with  5-­‐
fluorouracil  to  enhance  
cytotoxicity  in  the  treatment  of  GI  
cancers  
A.  N-­‐acetylcysteine  
 
B.  Mesna  
 
C.  Acrolein  
D.  Leucovorin  
220.  Hematologic  malignancy  
associated  with  an  increased  risk  
for  disseminated  intravascular  
coagulaLon    

A. Chronic  Myeloid  Leukemia    


B. Acute  PromyelocyLc  Leukemia    
C. Non-­‐Hodgkin’s  Lymphoma    
D. MulLple  myeloma    
220.  Hematologic  malignancy  
associated  with  an  increased  risk  
for  disseminated  intravascular  
coagulaLon    

A. Chronic  Myeloid  Leukemia    


B.  Acute  PromyelocyNc  Leukemia    
C. Non-­‐Hodgkin’s  Lymphoma    
D. MulLple  myeloma    

Vous aimerez peut-être aussi